Combined PrepU Kidney

Ace your homework & exams now with Quizwiz!

The laboratory results for a patient with renal failure, accompanied by decreased glomerular filtration, would be evaluated frequently. Which of the following is the most sensitive indicator of renal function?

serum creatinine 1.5

In a diagnosis of an upper urinary tract infection, which structures could be affected? Select all that apply.

ureter kidney

A client is admitted for treatment of chronic renal failure (CRF). The nurse knows that this disorder increases the client's risk of:

water and sodium retention secondary to a severe decrease in the glomerular filtration rate.

An instructor is preparing a class on renal cancer for a group of students. Which of the following would the instructor include as a possible risk factor?

age with most renal cancers occurring after age 60, male gender, tobacco use, occupational exposure to industrial chemicals, obesity, unopposed estrogen therapy, polycystic kidney disease, and treatment for renal failure.

The dialysate should be warmed in a

commercial warmer

The nurse admits a client post ileal conduit. What is the top priority?

Vital signs

A patient diagnosed AKI has a serum potassium level of 6.5 mEq/L. The nurse anticipates administering_____.

Kayexalate -cation-exchage resins. Exchanges sodium ions for potassium ions in the intestinal tract

A client with chronic renal failure comes to the clinic for a visit. During the visit, he complains of pruritus. Which suggestion by the nurse would be most appropriate?

Keep your showers brief, patting your skin dry after showering."

A 44-year-old client is in the hospital unit where you practice nursing. From the results of a series of diagnostic tests, she has been diagnosed with acute glomerulonephritis. What would you expect to find as a result of this condition?

Proteinuria

Azotemia

term used to describe excessive nitrogenous waster in the blood, as seen in acute glomerulonephritis

The nurse is reviewing a patient's laboratory results. What findings does the nurse assess that are consistent with acute glomerulonephritis? (Select all that apply.)

Red blood cells in the urine Proteinuria White cell casts in the urine

Sympathomimetics have which of the following effects on the body?

Relaxation of bladder wall

A patient has been diagnosed with postrenal failure. The nurse reviews the patient's electronic health record and notes a possible cause. Which of the following is the possible cause?

Renal calculi

Following a nephrectomy, which assessment finding is most important in determining nursing care for the client?

SpO2 at 90% with fine crackles in the lung bases

Which clinical finding should a nurse look for in a client with chronic renal failure?

Uremia

Postnephrectomy common pt challenges

Due to location of incision, the risk for ineffective breathing patter is common Maintain SpO2 levels at 90% or greater to keep lungs clear of adventitious sounds. -Blood tinged drainage from the JP tube is expected

Osteodystrophy

a condition in which the bone becomes demineralized due to hypocalcemia and hyperphosphatemia. In an effort to raise blood calcium levels, the parathyroid glands secrete more parathormone.

The client with polycystic kidney disease asks the nurse, "Will my kidneys ever function normally again?" The best response by the nurse is:

"As the disease progresses, you will most likely require renal replacement therapy."

A client with newly diagnosed renal cancer is questioning why detection was delayed. Which is the best response by the nurse? a) "Painless gross hematuria is the first symptom in renal cancer." b) "Squamous cell carcinomas do not present with detectable symptoms." c) "You should have sought treatment earlier." d) "Very few symptoms are associated with renal cancer."

"Very few symptoms are associated with renal cancer." Explanation: Renal cancers rarely cause symptoms in the early stage. Tumors can become quite large before causing symptoms. Painless, gross hematuria is often the first symptom in renal cancer and does not present until later stages of the disease. Adenocarcinomas are the most common renal cancer (about 80%),whereas squamous cell renal cancers are rare. It is not therapeutic to place doubt or blame for delayed diagnosis.

What is the normal urine output for the infant/child?

0.5 - 2ml/kg/hr

B

10. The nurse is caring for a patient after kidney surgery. What major danger should the nurse closely monitor for? a. Abdominal distention owing to reflex cessation of intestinal peristalsis b. Hypovolemic shock caused by hemorrhage c. Paralytic ileus caused by manipulation of the colon during surgery d. Pneumonia caused by shallow breathing because of severe incisional pain

The nurse is reviewing the results of a client's renal function study. The nurse understands that which value represent a normal BUN-to-creatinine ratio?

10:1

How many hours do you have to correct testicular torsion in order to save the testis?

6 hours

Which of the following diagnostic tests would the nurse expect to be ordered to determine the details of the arterial supply to the kidneys?

Angiography

When assessing the impact of medications on the etiology of acute renal failure, the nurse recognizes which of the following as the drug that is not nephrotoxic? A. Neomycin B. Penicillin

B. Penicillin

A patient with chronic kidney failure experiences decreased levels of erythropoietin. What serious complication related to those levels should the nurse assess for when caring for this client? A. Acidosis B. Pericarditis C. Anemia D. Hyperkalemia

C. Anemia

A client with acute renal failure progresses through four phases. Which describes the onset phase?

It is accompanied by reduced blood flow to the nephrons. R:The onset phase is accompanied by reduced blood flow to the nephrons. In the oliguric phase, fluid volume excess develops, which leads to edema, hypertension, and cardiopulmonary complications. During the diuretic phase, excretion of wastes and electrolytes continues to be impaired despite increased water content of the urine. During the recovery phase, normal glomerular filtration and tubular function are restored.

Which of the following is used to decrease potassium level seen in acute renal failure?

Kayexalate

The nurse is preparing a client for a nuclear scan of the kidneys. Following the procedure, the nurse instructs the client to

drink liberal amounts of fluids.

For a client in the oliguric phase of acute renal failure (ARF), which nursing intervention is the most important?

Limiting fluid intake

BPH s/s

hesitancy, decrease in size and force of urine stream, frequency, nocturia; retention- bladder sound will be dull on percussion

A client has been diagnosed with acute glomerulonephritis. This condition causes:

proteinuria.

A nurse is caring for an acutely ill patient. The nurse understands that the most accurate indicator of fluid loss or gain in an acutely ill patient is which of the following?

weight

The nurse recognizes which condition as an integumentary manifestation of chronic renal failure? A. Tremors B. Seizures C. Asterixis D. Gray-bronze skin color

D. Gray-bronze skin color

An appropriate nursing intervention for the client following a nuclear scan of the kidney is to:

Encourage high fluid intake.

What is an indicator of infection or transplant rejection?

Fever

A female patient undergoes dialysis as a part of treatment for kidney failure. The patient is administered heparin during dialysis to achieve therapeutic levels. Which of the following steps should the nurse take to allow heparin to be metabolized and excreted in the patient?

Avoid administering injections for 2 to 4 hours after heparin administration.

C

11. A patient undergoing a CT scan with contrast has a baseline creatinine level of 3 mg/dL, identifying this patient as at a high risk for developing kidney failure. What is the most effective intervention to reduce the risk of developing radiocontrast-induced nephropathy (CIN)? a. Performing the test without contrast b. Administering Garamycin (gentamicin) prophylactically c. Hydrating with saline intravenously before the test d. Administering sodium bicarbonate after the procedure

A child diagnosed with pyelonephritis has a fever of 104.2. Which antipyretic should the nurse administer?

Acetaminophen

What does an increased BUN indicate?

Acute Renal failure

A 24-hour urine collection is scheduled to begin at 8:00 am. When should the nurse initiate the procedure?

After discarding the 8:00 am specimen

The client with chronic renal failure complains of intense itching. Which assessment finding would indicate the need for further nursing education?

Brief hot daily showers Hot water removes more oils from the skin and can increase dryness and itching.

The client with chronic renal failure is exhibiting signs of anemia. Which is the best nursing rationale for this symptom? A. Impaired immunologic response B. Azotemia C. Diminished erythropoietin production D. Electrolyte imbalances

C. Diminished erythropoietin production

What is an organic cause for erectile dysfunction?

Coronary artery disease

Which of the following nursing actions is most important in caring for the client following lithotripsy?

Strain the urine carefully for stone fragments.

The nurse is caring for a client who is brought to the emergency department after being found unconscious outside in hot weather. Dehydration is suspected. Baseline lab work including a urine specific gravity is ordered. Which relation between the client's symptoms and urine specific gravity is anticipated?

The specific gravity will be high.

The nurse expects which of the following assessment findings in the client in the diuretic phase of acute renal failure?

Dehydration

Which phase of acute renal failure signals that glomerular filtration has started to recover?

Diuretic

integumentary manifestation of chronic renal failure?

Gray-Bronze skin color

The nurse recognizes which condition as an integumentary manifestation of chronic renal failure?

Gray-bronze skin color

The nurse passes out medications while a client prepares for hemodialysis. The client is ordered to receive numerous medications including antihypertensives. What is the best action for the nurse to take?

Hold the medications until after dialysis.

Which problems identified during prenatal care could represent potential renal problems during childhood?

Hydronephrosis Neural tube defect Oligohydramnios One umbilical artery

The nurse is caring for a patient in acute renal failure. The nurse should expect hypertonic glucose, insulin infusions, and sodium bicarbonate to be used to treat:

Hyperkalemia

Which assessment is a symptom of nephrosclerosis?

Hypertension

A persistent painful erection is called....

Praprism

What is the term for the concentration of urea and other nitrogenous wastes in the blood? a) Uremia b) Azotemia c) Proteinuria d) Hematuria

Azotemia Explanation: Azotemia is the concentration of urea and other nitrogenous wastes in the blood. Uremia is an excess of urea and other nitrogenous wastes in the blood. Hematuria is blood in the urine. Proteinuria is protein in the urine.

A

13. The nurse is administering calcium acetate (PhosLo) to a patient with ESKD. When is the best time for the nurse to administer this medication? a. With food b. 2 hours before meals c. 2 hours after meals d. At bedtime with 8 ounces of fluid

The nurse is providing discharge instructions to the client with acute post-streptococcal glomerulonephritis. Which statement by the client indicates a need for further teaching?

"I should drink as much as possible to keep my kidneys working." R: fluid restriction is a tx of this problem

Which type of incontinence refers to involuntary loss of urine through an intact urethra as a result of a sudden increase in intra-abdominal pressure?

Stress

Priapism can also be caused by what other process?

Sickle cell disease

What is the action of Phosphodiesterase inhibitors?

Smooth muscle relaxation

A nurse is reviewing the laboratory test results of a client with renal disease. Which of the following would the nurse expect to find?

Increased serum creatinine

Mr. Billings is being seen as a client in the urology practice. He has a family history of polycystic kidney disease. Of the following assessment findings, which would you expect to find as you gather information and complete an assessment related to a polycystic kidney diagnosis? Select all that apply. a) Hypertension b) Normal urinalysis c) No renal stones d) Pain from retroperitoneal bleeding

• Hypertension • Pain from retroperitoneal bleeding Explanation: Hypertension is present in approximately 75% of affected clients at the time of diagnosis. Pain from retroperitoneal bleeding is caused by the size and effects of the cysts. Urinalysis shows mild proteinuria, hematuria, and pyuria. Renal stones are common.

A client admitted with a gunshot wound to the abdomen is transferred to the intensive care unit after an exploratory laparotomy. I.V. fluid is being infused at 150 ml/hour. Which assessment finding suggests that the client is experiencing acute renal failure (ARF)? a) Urine output of 250 ml/24 hours b) Temperature of 100.2° F (37.8° C) c) Serum creatinine level of 1.2 mg/dl d) Blood urea nitrogen (BUN) level of 22 mg/dl

A) Urine output of 250 ml/24 hours ARF, characterized by abrupt loss of kidney function, commonly causes oliguria, which is characterized by a urine output of 250 ml/24 hours. A serum creatinine level of 1.2 mg/dl isn't diagnostic of ARF. A BUN level of 22 mg/dl or a temperature of 100.2° F (37.8° C) wouldn't result from this disorder.

A client is experiencing some secretion abnormalities, for which diagnostics are being performed. Which substance is typically reabsorbed and not secreted in urine?

glucose

A nurse is reviewing the history of a client who is suspected of having glomerulonephritis. Which of the following would the nurse consider significant?

group A beta-hemolytic streptococcal infections, bacterial endocarditis, or viral infections such as hepatitis B or C or human immunodeficiency virus (HIV).

A client develops acute renal failure (ARF) after receiving I.V. therapy with a nephrotoxic antibiotic. Because the client's 24-hour urine output totals 240 ml, the nurse suspects that the client is at risk for:

cardiac arrhythmia.

The nurse is reviewing the potassium level of a patient with kidney disease. The results of the test are 6.5 mEq/L, and the nurse observes peaked T waves on the ECG. What priority intervention does the nurse anticipate the physician will order to reduce the potassium level?

give kayexalate

When assessing the impact of medications on the etiology of ARF, the nurse recognizes which of the following as the drug that is not nephrotoxic?

penicillin

A client who suffered hypovolemic shock during a cardiac incident has developed acute renal failure. Which is the best nursing rationale for this complication? A. Decrease in the blood flow through the kidneys B. Blood clot formed in the kidneys interfered with the flow C. Structural damage occurred in the nephrons of the kidneys D. Obstruction of urine flow from the kidneys

A. Decrease in the blood flow through the kidneys

What is used to decrease potassium level seen in acute renal failure? A. Sodium polystyrene sulfonate B. Calcium supplements C. IV dextrose 50% D. Sorbitol

A. Sodium polystyrene sulfonate

A history of infection specifically caused by group A beta-hemolytic streptococci is associated with which disorder?

Acute glomerulonephritis

Your client has a diagnosis of hypervolemia. What would be an important intervention that you would initiate? a) Give medications that promote fluid retention. b) Limit sodium and water intake. c) Teach client behaviors that decrease urination. d) Assess for dehydration.

B) Limit sodium and water intake Implement prescribed interventions such as limiting sodium and water intake and administering ordered medications that promote fluid elimination. Assessing for dehydration and teaching to decrease urination would not be appropriate interventions.

A patient is being seen in the clinic for possible kidney disease. What major sensitive indicator of kidney disease does the nurse anticipate the patient will be tested for?

Creatinine clearance level

A client who is blind is admitted for treatment of gastroenteritis. Which nursing diagnosis takes highest priority for this client?

Deficient fluid volume

An elderly client is being evaluated for suspected pyelonephritis and is ordered kidney, ureter, and bladder (KUB) x-ray. The nurse understands the significance of this order is related to which rationale?

Detects calculi, cysts, or tumors

Ms. Linden is in end-stage chronic renal failure and is being added to the transplant list. You are explaining to her how donors are found for clients needing kidneys. You would be accurate in telling her which of the following? a) Donors with hypertension may qualify. b) Donors are selected from compatible living donors. c) The client is placed on a transplant list at the local hospital. d) Donors must be relatives.

Donors are selected from compatible living donors. Explanation: Donors are selected from compatible living donors. Donors do not have to be relatives as long as they are compatible. Potential donors with a history of hypertension, malignant disease, or diabetes are excluded from donation. The client is placed on a national computerized transplant waiting list.

A group of students are reviewing the phases of acute renal failure. The students demonstrate understanding of the material when they identify which of the following as occurring during the second phase?

During the second phase, the oliguric phase, oliguria occurs. Diuresis occurs during the third or diuretic phase. Acute tubular necrosis (ATN) occurs during the first, or initiation, phase in which reduced blood flow to the nephrons leads to ATN. Restoration of glomerular function, if it occurs, occurs during the fourth, or recovery, phase.

A school nurse is trying to prevent poststreptococcal glomerulonephritis in children. Which of the following would be the best way to prevent this?

Encourage the child to take all the antibiotics if diagnosed with strep throat.

A client, aged 87, undergoes continuous ambulatory peritoneal dialysis (CAPD) for acute renal failure (ARF). Which task would be most important for the nurse to do?

Frequently monitor the client's progress. R:Older clients who are not candidates for kidney transplants may receive CAPD. More frequent monitoring of the client's progress is required when this technique is used. The recommendations for protein and potassium in the diet are highly variable based on the client's condition. Change in the color of client's teeth, eyes, and nails need not be monitored, nor does the client need to be monitored for hypoglycemia and hyperglycemia.

The nurse is collecting data on a 2-year-old child admitted with a diagnosis of urinary tract infection. When interviewing the caregivers, which of the following questions would be most important for the nurse to ask?

Gather information about the current illness: when the fever started and its course thus far, signs of pain or discomfort on voiding, recent change in feeding pattern, presence of vomiting or diarrhea, irritability, lethargy, abdominal pain, unusual odor to urine, chronic diaper rash, and signs of febrile convulsions. Toilet training and bathing habits would be of importance, but they are not the most important to ask.

Which of the following is an integumentary manifestation of chronic renal failure?

Gray-bronze skin color. Other manifestations are dry, flaky skin, pruritus, ecchymosis, purpura, thin, brittle nails, and coarse, thinning hair. Asterixis, tremors, and seizures are neurologic manifestations of chronic renal failure.

Which of the following is as integumentary manifestation of chronic renal failure?

Gray-brown skin color Integumentary manifestations of chronic renal failure include a gray-bronze skin color and ecchymosis.

Which of the following would be included in the plan of care of someone in Stage 4 renal failure?

Growth hormone injections daily

The nurse passes out medications while a client prepares for hemodialysis. The client is ordered to receive numerous medications including antihypertensives. What is the best action for the nurse to take?

Hold meds till after dialysis R: Specifically BP meds so it doesn't drop too low during the fluid exchange. Also once a day meds can often be held till after tx.

The nurse is passing out medications on a medical-surgical unit. A male patient is preparing for hemodialysis. The patient is ordered to receive numerous medications including antihypertensives. Which of the following is the best action for the nurse to take? a) Hold the medications until after dialysis. b) Ask the patient if he wants to take his medications. c) Administer the medications as ordered. d) Check with the dialysis nurse about the medications.

Hold the medications until after dialysis. Explanation: Antihypertensive therapy, often part of the regimen of patients on dialysis, is one example when communication, education, and evaluation can make a difference in patient outcomes. The patient must know when—and when not—to take the medication. For example, if an antihypertensive agent is taken on a dialysis day, hypotension may occur during dialysis, causing dangerously low blood pressure. Many medications that are taken once daily can be held until after dialysis treatment.

A patient undergoing a CT scan with contrast has a baseline creatinine level of 3 mg/dL, identifying this patient as at a high risk for developing kidney failure. What is the most effective intervention to reduce the risk of developing radiocontrast-induced nephropathy (CIN)? a) Hydrating with saline intravenously before the test b) Performing the test without contrast c) Administering Garamycin (gentamicin) prophylactically d) Administering sodium bicarbonate after the procedure

Hydrating with saline intravenously before the test Explanation: Radiocontrast-induced nephropathy (CIN) is a major cause of hospital-acquired AKI. This is a potentially preventable condition. Baseline levels of creatinine greater than 2 mg/dL identify patients at high risk. Limiting the patient's exposure to contrast agents and nephrotoxic medications will reduce the risk of CIN. Administration of N-acetylcysteine and sodium bicarbonate before and during procedures reduces risk, but prehydration with saline is considered the most effective method to prevent CIN

The nurse is caring for a patient in acute renal failure. The nurse should expect hypertonic glucose, insulin infusions, and sodium bicarbonate to be used to treat:

Hyperkalemia. R:Hyperkalemia is a common complication of acute renal failure. It is life-threatening if immediate action isn't taken to reverse it. The administration of glucose and regular insulin, with sodium bicarbonate if necessary, can temporarily prevent cardiac arrest by moving potassium into the cells and temporarily reducing serum potassium levels. Hypernatremia, hypokalemia, and hypercalcemia don't usually occur with acute renal failure and aren't treated with glucose, insulin, or sodium bicarbonate.

A chronic renal failure client complains of generalized bone pain and tenderness. Which assessment finding would alert the nurse to an increased potential for the development of spontaneous bone fractures? a) Elevated serum creatinine b) Hyperkalemia c) Hyperphosphatemia d) Elevated urea and nitrogen

Hyperphosphatemia Explanation: Osteodystrophy is a condition in which the bone becomes demineralized due to hypocalcemia and hyperphosphatemia. In an effort to raise blood calcium levels, the parathyroid glands secrete more parathormone. Elevated creatinine, urea, nitrogen, and potassium levels are expected in chronic renal failure and do not contribute to bone fractures.

A chronic renal failure client complains of generalized bone pain and tenderness. Which assessment finding would alert the nurse to an increased potential for the development of spontaneous bone fractures?

Hyperphosphatemia. Osteodystrophy is a condition in which the bone becomes demineralized due to hypocalcemia and hyperphosphatemia. In an effort to raise blood calcium levels, the parathyroid glands secrete more parathormone.

A client is diagnosed with polycystic kidney disease. Which of the following would the nurse most likely assess? a) Periorbital edema b) Extremity pain c) Hypertension d) Fever

Hypertension Explanation: Hypertension is present in approximately 75% of clients with polycystic kidney disease at the time of diagnosis. Pain from retroperitoneal bleeding, lumbar discomfort, and abdominal pain also may be noted based on the size and effects of the cysts. Fever would suggest an infection. Periorbital edema is noted with acute glomerulonephritis.

A patient arrives at the ED via ambulance following a motor cycle accident. The paramedics state the patient was found unconscious at the scene of the accident, but briefly regained consciousness during transport to the hospital. Upon initial assessment, the patient's GCS score is 7. The nurse anticipates which of the following? a) Intubation and mechanical ventilation b) An order for a head CT scan c) Immediate craniotomy d) Administration of propofol (Diprivan) IV

Immediate craniotomy Explanation: The patient is experiencing an epidural hematoma. An epidural hematoma is considered an extreme emergency; marked neurologic deficit or even respiratory arrest can occur within minutes. Treatment consists of making openings through the skull (burr holes) to decrease ICP emergently, remove the clot, and control the bleeding. A craniotomy may be required to remove the clot and control the bleeding. Epidural hematomas are often characterized by a brief loss of consciousness followed by a lucid interval in which the patient is awake and conversant. During this lucid interval, compensation for the expanding hematoma takes place by rapid absorption of CSF and decreased intravascular volume, both of which help to maintain the ICP within normal limits. When these mechanisms can no longer compensate, even a small increase in the volume of the blood clot produces a marked elevation in ICP. The patient then becomes increasingly restless, agitated, and confused as the condition progresses to coma.

The main problem with acute glomerulonephritis is ....

Inflammation

A client is administered dialysate solution through an abdominal catheter. The nurse notices that the return flow rate is slow, so the nurse advises the client to move to the other side. However, even after changing the client's position, the nurse does not observe an increase in return flow. Which of the following actions should the nurse perform to help accelerate the return flow rate?

Inform the physician that catheter may need repositioning. R:The nurse instills dialysate solution and clamps the tubing. If the return flow rate is slow, the nurse asks the client to move from side to side. If this maneuver is unsuccessful, the physician may need to reposition the catheter. The nurse should not tamper with the catheter settings, as this may worsen the client's condition or damage the apparatus. Also, stopping the process and resuming it after 5 minutes will not help increase the return flow of the dialysate.

A patient diagnosed AKI has a serum potassium level of 6.5 mEq/L. The nurse anticipates administering which of the following?

Kayexolate

The nurse helps a client to correctly perform peritoneal dialysis at home. The nurse must educate the client about the procedure. Which educational information should the nurse provide to the client?

Keep the dialysis supplies in a clean area, away from children and pets

A client diagnosed with acute kidney injury (AKI) has developed congestive heart failure. The client has received 40 mg of intravenous push (IVP) Lasix and 2 hours later, the nurse notes that there are 50 mL of urine in the Foley catheter bag. The client's vital signs are stable. Which health care order should the nurse anticipate?

Lasix 80 mg IVP R: Diuretic agents are often used to control fluid volume in clients with acute kidney injury (AKI). The client's urine output indicates an inadequate response to the initial dosage of Lasix and the nurse should anticipate administering Lasix 80 mg IVP. Often in this situation, the initial dosage of Lasix is doubled. The client is experiencing fluid overload, thus, a 500-mL bolus of normal saline bolus would be contraindicated. There is no need to complete a chest x-ray. Mannitol is widely used in the management of cerebral edema and increased intracranial pressure from multiple causes.

The nurse is caring for a patient in the oliguric phase of acute kidney injury (AKI). What does the nurse know would be the daily urine output?

Less than 400 mL

The nurse is caring for a patient in the oliguric phase of AKI. What does the nurse know would be the daily urine output? a) 1.5 L b) Less than 400 mL c) Less than 50 mL d) 1.0 L

Less than 400 mL Explanation: The oliguria period is accompanied by an increase in the serum concentration of substances usually excreted by the kidneys (urea, creatinine, uric acid, organic acids, and the intracellular cations [potassium and magnesium]). The minimum amount of urine needed to rid the body of normal metabolic waste products is 400 mL. In this phase, uremic symptoms first appear and life-threatening conditions such as hyperkalemia develop.

The nurse performs a physical examination on a client diagnosed with acute pyelonephritis to assist in determining which of the following?

Location of discomfort

Which of the following is the earliest sign of increasing intracranial pressure (ICP)? a) Headache b) Vomiting c) Posturing d) Loss of consciousness

Loss of consciousness Explanation: The earliest sign of increasing ICP is loss of consciousness. Other manifestations of increasing ICP are vomiting, headache, and posturing.

Which of the following would a nurse classify as a prerenal cause of acute renal failure?

Prerenal causes of acute renal failure include hypovolemic shock, cardiogenic shock secondary to congestive heart failure, septic shock, anaphylaxis, dehydration, renal artery thrombosis or stenosis, cardiac arrest, and lethal dysrhythmias. Ureteral stricture and prostatic hypertrophy would be classified as postrenal causes. Polycystic disease is classified as an intrarenal cause of acute renal failure.

A patient sustained a head injury and has been admitted to the neurosurgical intensive care unit (ICU). The patient began having seizures and was administered a sedative-hypnotic medication that is ultra-short acting and can be titrated to patient response. What medication will the nurse be monitoring during this time? a) Midazolam (Versed) b) Lorazepam (Ativan) c) Phenobarbital d) Propofol (Diprivan)

Propofol (Diprivan) Explanation: If the patient is very agitated, benzodiazepines are the most commonly used sedative agents and do not affect cerebral blood flow or ICP. Lorazepam (Ativan) and midazolam (Versed) are frequently used but have active metabolites that my cause prolonged sedation, making it difficult to conduct a neurologic assessment. Propofol ( Diprivan), on the other hand, a sedative-hypnotic agent that is supplied in an intralipid emulsion for intravenous (IV) use, is the sedative of choice. It is an ultra-short acting, rapid onset drug with elimination half-life of less than an hour. It has a major advantage of being titratable to its desired clinical effect but still provides the opportunity for an accurate neurologic assessment (Hickey, 2009).

Which of the following is the hallmark of the diagnosis of nephrotic syndrome?

Proteinuria

The nurse is caring for a client immediately after a spinal cord injury. Which assessment finding is essential when caring for a client in spinal shock with injury in the lower thoracic region? a) Pain level b) Numbness and tingling c) Pulse and blood pressure d) Respiratory pattern

Pulse and blood pressure Explanation: Spinal shock is a loss of sympathetic reflex activity below the level of the injury within 30 to 60 minutes after insult. In addition to the paralysis, manifestations include pronounced hypotension, bradycardia, and warm, dry skin. Numbness and tingling and pain are not as high of a concern at this time due to the cord injury. Because the level of impairment is below the first thoracic vertebrae, respiratory failure is not a concern.

The nurse is reviewing a patient's laboratory results. What findings does the nurse assess that are consistent with acute glomerulonephritis? Select all that apply.

Red blood cells in the urine Proteinuria White cell casts in the urine R: The primary presenting features of an acute glomerular inflammation are hematuria, edema, azotemia (an abnormal concentration of nitrogenous wastes in the blood), and proteinuria (excess protein in the urine) (Porth & Matfin, 2009). The urine may appear cola colored because of red blood cells (RBCs) and protein plugs or casts; RBC casts indicate glomerular injury.

The nurse is treating a patient with ESKD. The nurse is concerned that the patient is developing renal osteodystrophy. Upon review of the patient's laboratory values, it is noted the patient has had a calcium level of 11 mg/dL for the past 3 days and the phosphate level is 5.5 mg/dL. The nurse anticipates the administration of which of the following medications? a) Mylanta b) Phos-Lo (calcium carbonate) c) Os-Cal (calcium carbonate) d) Renagel (sevelamer)

Renagel (sevelamer) Explanation: Hyperphosphatemia and hypocalcemia are treated with medications that bind dietary phosphorus in the GI tract. Binders such as calcium carbonate (Os-Cal) or calcium acetate (PhosLo) are prescribed, but there is a risk of hypercalcemia. If calcium is high or the calcium-phosphorus product exceeds 55 mg/dL, a polymeric phosphate binder such as sevelamer hydrochloride (Renagel) may be prescribed. This medication binds dietary phosphorus in the intestinal tract; one to four tablets should be administered with food to be effective. Magnesium-based antacids are avoided to prevent magnesium toxicity.

A nurse is caring for a 16-year-old adolescent with a head injury resulting from a fight after a high school football game. A physician has intubated the client and written orders to wean him from sedation therapy. A nurse needs further assessment data to determine whether: a) to continue I.V. administration of other scheduled medications. b) nutritional protocol will be effective after the client sedation therapy is tapered. c) she'll have to apply restraints to prevent the client from dislodging the endotracheal (ET) tube. d) payment status will change if the client isn't sedated.

She'll have to apply restraints to prevent the client from dislodging the endotracheal (ET) tube. Explanation: When the client isn't sedated, he may make attempts to remove the ET tube without realizing what he's doing. The nurse needs to obtain information to determine whether it's necessary to request an order for restraints. The nurse doesn't need to obtain additional data to determine if the nutritional protocol will continue to reflect the client's needs because this aspect of care won't change. The client doesn't require additional assessments to continue I.V. administration of medications. I.V. medication clearly needs to continue because the client is intubated. The staff nurse doesn't need to monitor payment status because client sedation shouldn't affect payment status.

A nurse assesses a client shortly after living donor kidney transplant surgery. Which postoperative finding must the nurse report to the physician immediately? a) Temperature of 99.2° F (37.3° C) b) Serum potassium level of 4.9 mEq/L c) Urine output of 20 ml/hour d) Serum sodium level of 135 mEq/L

Urine output of 20 ml/hour Explanation: Because kidney transplantation carries the risk of transplant rejection, infection, and other serious complications, the nurse should monitor the client's urinary function closely. A decrease from the normal urine output of 30 ml/hour is significant and warrants immediate physician notification. A serum potassium level of 4.9 mEq/L, a serum sodium level of 135 mEq/L, and a temperature of 99.2° F are normal assessment findings.

Because of difficulties with hemodialysis, peritoneal dialysis is initiated to treat a client's uremia. Which finding during this procedure signals a significant problem?

White blood cell (WBC) count of 20,000/mm3

Because of difficulties with hemodialysis, peritoneal dialysis is initiated to treat a client's uremia. Which finding during this procedure signals a significant problem?

White blood cell (WBC) count of 20,000/mm3. An increased WBC count indicates infection, probably resulting from peritonitis, which may have been caused by insertion of the peritoneal catheter into the peritoneal cavity. Peritonitis can cause the peritoneal membrane to lose its ability to filter solutes; therefore, peritoneal dialysis would no longer be a treatment option for this client. Hyperglycemia (evidenced by a blood glucose level of 200 mg/dl) occurs during peritoneal dialysis because of the high glucose content of the dialysate; it's readily treatable with sliding-scale insulin. A potassium level of 3.5 mEq/L can be treated by adding potassium to the dialysate solution.

dialysis equilibrium syndrome

a complication that's most common during the first few dialysis sessionsCauses confusion, decreasing level of consciousness, headache, and seizures. May last several days

A client with acute renal failure is undergoing dialysis for the first time. The nurse monitors the client closely for dialysis equilibrium syndrome, a complication that's most common during the first few dialysis sessions. Typically, dialysis equilibrium syndrome causes:

confusion, headache, and seizures.

patient teaching for pruritis

skin clean and dry brief showers with tepid water pat the skin dry use moisturizing lotions /creams avoid scratching use a mild laundry detergent add extra rinse cycle add 1 tsp vinegar per quart of water to the rinse cycle to remove detergent residue.

A nurse assesses a patient diagnosed in the prerenal stage of ARF. The nurse expects to find the following signs and symptoms. Select all that apply. a) Urine specific gravity of 1.029 b) Urine sodium <20 mEq/L c) Urine osmolality of 350 mOsm/Kg d) BUN value of <10 mg/dL e) Creatinine level of 1.3 mg/dL f) Increase in urinary sediment

• Creatinine level of 1.3 mg/dL • Urine sodium <20 mEq/L • Urine specific gravity of 1.029 Explanation: The BUN reading is within normal range, the urine osmolality would be greater than 500 mOsm/kg, and there would be few hyaline casts. Refer to Table 27-2 in the text.

The nurse is reviewing a patient's laboratory results. What findings does the nurse assess that are consistent with acute glomerulonephritis? (Select all that apply.) a) Red blood cells in the urine b) Polyuria c) White cell casts in the urine d) Proteinuria e) Hemoglobin of 12.8 g/dL

• Red blood cells in the urine • Proteinuria • White cell casts in the urine Explanation: The primary presenting features of an acute glomerular inflammation are hematuria, edema, azotemia (an abnormal concentration of nitrogenous wastes in the blood), and proteinuria (excess protein in the urine). The urine may appear cola colored because of red blood cells (RBCs) and protein plugs or casts; RBC casts indicate glomerular injury.

Elevated ICP is most commonly associated with head injury. Which of the following are clinical signs of increased ICP that a nurse should evaluate? Select all that apply. a) Slow bounding pulse b) Widened pulse pressure c) Increased cerebral perfusion d) Respiratory irregularities e) Lowered systolic blood pressure

• Slow bounding pulse • Widened pulse pressure • Respiratory irregularities Explanation: In the early stages of cerebral ischemia, the vasomotor centers are stimulated and the systemic pressure rises to maintain cerebral blood flow. This is typically accompanied by a slow, bounding pulse and respiratory irregularities. These changes in blood pressure, pulse, and respiration are important clinically because they suggest increased ICP. A sympathetically mediated response causes an increase in the systolic blood pressure, with a widening of the pulse pressure and cardiac slowing.

When caring for a child who has a urinary tract infection, which of the following nursing interventions would be most appropriate. (select all that apply)

• The nurse observes for signs of pain or burning on urination. • The nurse monitors intake and output. • The nurse administers antipyretics as needed.

Which of the following is a term used to describe excessive nitrogenous waster in the blood, as seen in acute glomerulonephritis?

Azotemia

Which term is used to describe the concentration of urea and other nitrogenous wastes in the blood?

Azotemia

D

12. The nurse is reviewing the potassium level of a patient with kidney disease. The results of the test are 6.5 mEq/L, and the nurse observes peaked T waves on the ECG. What priority intervention does the nurse anticipate the physician will order to reduce the potassium level? a. Administration of an insulin drip b. Administration of a loop diuretic c. Administration of sodium bicarbonate d. Administration of sodium polystyrene sulfonate [Kayexalate])

D

14. A patient with ESKD is scheduled to have a arteriovenous fistula created. The nurse explains that the patient will have a temporary dialysis catheter because the fistula has to "mature." The nurse will explain that the patient will have to wait how long before using the fistula? a. 1 to 2 weeks b. 2 to 3 weeks c. 1 month d. 2 to 3 months

To determine if ascites is increasing in amount in a child with nephrotic syndrome, which measurements would be most appropriate?

Abdominal circumference

A client develops decreased renal function and requires a change in antibiotic dosage. On which factor should the physician base the dosage change?

Creatinine clearance

Which value does the nurse recognize as the best clinical measure of renal function?

Creatinine clearance

The nurse is providing discharge instructions to the client with acute post-streptococcal glomerulonephritis. Which statement by the client indicates a need for further teaching?

"I should drink as much as possible to keep my kidneys working." R:Dietary management of acute post-streptococcal glomerulonephritis includes restrictions of protein, sodium, potassium, and fluids.

A client who has been treated for chronic renal failure (CRF) is ready for discharge. The nurse should reinforce which dietary instruction?

"Increase your carbohydrate intake."

After teaching a group of students about how to perform peritoneal dialysis, which statement would indicate to the instructor that the students NEED additional teaching?

"It is appropriate to warm the dial sate in a microwave."

A nurse is interviewing the parents of a child diagnosed with obstructive uropathy. Which statement by the parents would the nurse identify as significant?

"She had surgery to repair a problem with her anus.". Risk factors associated with obstructive uropathy include prune belly syndrome, chromosome abnormalities, anorectal malformations, and ear defects.

Which statement by the client with end-stage renal disease indicates teaching by the nurse was effective?

"Ultrafiltration methods take much longer than hemodialysis."

Which statement by the client with end-stage renal disease indicates teaching by the nurse was effective? a) "Ultrafiltration methods take much longer than hemodialysis." b) "There are few complications with renal replacement therapies." c) "A family member can help me perform hemodialysis in my home." d) "A special access is created in my vein for peritoneal dialysis."

"Ultrafiltration methods take much longer than hemodialysis." Explanation: Ultrafiltration methods (CVVH, CVVHD) are better tolerated by unstable clients as fluid is removed slowly, resulting in mild hemodynamic effects.

The nurse notes that a patient who is retaining fluid had a 1-kg weight gain. The nurse knows that this is equivalent to about how many mL?

1,000 mL

1-kg weight gain is equal to

1,000 mL of retained fluid

1-kg weight gain is equivalent to how many mL?

1,000mL

The nurse notes that a patient who is retaining fluid had a 1-kg weight gain. The nurse knows that this is equivalent to about how many mL?

1,000mL OR 1 L

D

1. The nurse notes that a patient who is retaining fluid had a 1-kg weight gain. The nurse knows that this is equivalent to about how many mL? a. 250 mL b. 500 mL c. 750 mL d. 1,000 mL

At the end of five peritoneal exchanges, a patient's fluid loss was 500 mL. How much is this loss equal to? a) 1.5 lb b) 1.0 lb c) 0.5 lb d) 2 lb

1.0 lb Explanation: The most accurate indicator of fluid loss or gain in an acutely ill patient is weight. An accurate daily weight must be obtained and recorded. A 1-kg (2-lb) weight loss is equal to 1,000 mL.

At the end of five peritoneal exchanges, a patient's fluid loss was 500 mL. How much is this loss equal to? a) 1.5 lb b) 2 lb c) 1.0 lb d) 0.5 lb

1.0 lb Explanation: The most accurate indicator of fluid loss or gain in an acutely ill patient is weight. An accurate daily weight must be obtained and recorded. A 1-kg (2-lb) weight loss is equal to 1,000 mL.

A

2. A patient admitted with electrolyte imbalance has carpopedal spasm, ECG changes, and a positive Chvostek's sign. What deficit does the nurse suspect the patient has? a. Calcium b. Magnesium c. Phosphorus d. Sodium

A, C, D

3. The nurse is reviewing a patient's laboratory results. What findings does the nurse assess that are consistent with acute glomerulonephritis? (Select all that apply.) a. Red blood cells in the urine b. Polyuria. c. Proteinuria d. White cell casts in the urine e. Hemoglobin of 12.8 g/dL

C

4. The nurse is caring for a patient in the oliguric phase of AKI. What does the nurse know would be the daily urine output? a. 1.5 L b. 1.0 L c. Less than 400 mL d. Less than 50 mL

The nurse is caring for a client with chronic kidney disease. The patient has gained 4 kg in the past 3 days. In milliliters, how much fluid retention does this equal? __________________

4000

B

5. The nurse is educating a patient who is required to restrict potassium intake. What foods would the nurse suggest the patient eliminate that are rich in potassium? a. Butter b. Citrus fruits c. Cooked white rice d. Salad oils

Sevelamer hydrochloride (Renagel) has been prescribed for a client with chronic renal failure. The physician has prescribed Renagel 800 mg orally three times per day with meals to treat the client's hyperphosphatemia. The medication is available in 400 mg tablets. How many tablets per day will the nurse administer to the client?

6 tablets

B

7. A patient has stage 3 chronic kidney failure. What would the nurse expect the patient's glomerular filtration rate (GFR) to be? a. A GFR of 90 mL/min/1.73 m2 b. A GFR of 30-59 mL/min/1.73 m2 c. A GFR of 120 mL/min/1.73 m2 d. A GFR of 85 mL/min/1.73 m2

A client who has a history of chronic renal failure is in stage 4 for CRF. What is the approximate level of nephron function loss?

75% to 90%

A

8. A patient with chronic kidney failure experiences decreased levels of erythropoietin. What serious complication related to those levels should the nurse assess for when caring for this patient? a. Anemia b. Acidosis c. Hyperkalemia d. Pericarditis

B

9. At the end of five peritoneal exchanges, a patient's fluid loss was 500 mL. How much is this loss equal to? a. 0.5 lb b. 1.0 lb c. 1.5 lb d. 2 lb

A client has undergone a renal transplant and returns to the health care agency for a follow-up evaluation. Which finding would lead to the suspicion that the client is experiencing rejection? a) Polyuria b) Abdominal pain c) Weight loss d) Hypotension

Abdominal pain Explanation: Signs and symptoms of transplant rejection include abdominal pain, hypertension, weight gain, oliguria, edema, fever, increased serum creatinine levels, and swelling or tenderness over the transplanted kidney site.

A patient has stage 3 chronic kidney failure. What would the nurse expect the patient's glomerular filtration rate (GFR) to be?

A GFR of 30-59 mL/min/1.73 m2

The nurse is completing a full exam of the renal system. Which assessment finding best documents the need to offer the use of the bathroom?

A dull sound when percussing over the bladder

Acute glomerulonephritis

A hx of infection specifically caused by group A beta-hemolytic streptococci

A 4-year-old girl with a urinary tract infection is scheduled to have a voiding cystourethrogram. When preparing her for this procedure, you would want to prepare her to

A voiding cystourethrogram requires the child to void during the procedure so that bladder emptying and urethra flow can be assessed.

The nurse expects which of the following assessment findings in the client in the diuretic phase of acute renal failure? a) Dehydration b) Crackles c) Hypertension d) Hyperkalemia

A) Dehydration The diuretic phase of acute renal failure is characterized by increased urine output, hypotension, and dehydration.

A patient with an obstruction of the renal artery causing renal ischemia exhibits HTN. One factor that may contribute to HTN: a) increase renin release b) increased ADH secretion c) decreased aldosterone secretion d) increased synthesis and release of prostaglandins

A) Increase Renin Release Renin is released in resonse to decreased B/P, renal ischemia, eosinophil chemotactic factor (ECF) depletion, and other factors affecting blood suppy to the kidney. It is they catalyst of the renin-angiotensin-aldosterone system, which raises B/P when stimulated. ADH is secreted by the posterior pituitary in response to serum hyperosmolality and low blood volume. Aldosterone is secreted within the renin-angiotensin II, and kidney prostaglandins lower B/P by causing vasodilation.

Retention of which electrolyte is the most life-threatening effect of renal failure? a) Potassium b) Calcium c) Phosphorous d) Sodium

A) Potassium Retention of potassium is the most life-threatening effect of renal failure.

A 32-year-old flight attendant is undergoing diagnostics due to a significant drop in renal output. The physician has scheduled an angiography and you are in the midst of completing client education about the procedure. The client asks what the angiography will reveal. What is your response, as her nurse? a) Renal circulation b) Urine production c) Kidney function d) Kidney structure

A) Renal circulation A renal angiogram (renal arteriogram) provides details of the arterial supply to the kidneys, specifically the location and number of renal arteries (multiple vessels to the kidney are not unusual) and the patency of each renal artery.

Which of the following would the nurse expect to find when reviewing the laboratory test results of a client with renal failure? A. Increased serum creatinine level B. Increased serum calcium level C. Decreased serum potassium level D. Increased red blood cell count

A. Increased serum creatinine level

All EXCEPT which patient teaching topic are appropriate to include for a patient with nephrosclerosis?

A. Low cholesterol, low sodium diet B. Smoking cessation C. Control blood glucose **D. Low calcium diet

A client diagnosed with acute kidney injury (AKI) has a serum potassium level of 6.5 mEq/L. The nurse anticipates administering: A. sodium polystyrene sulfonate (Kayexalate) B. Sorbitol C. IV dextrose 50% D. Calcium supplements

A. sodium polystyrene sulfonate (Kayexalate)

A history of infection specifically caused by group A beta-hemolytic streptococci is associated with which of the following disorders?

Acute glomerulonephritis- also associated with varicella zoster virus, hepatitis B, and Epstein-Barr virus. Acute renal failure- associated with hypoperfusion to the kidney, parenchymal damage to the glomeruli or tubules, and obstruction at a point distal to the kidney. Chronic renal failure- may be caused by systemic disease, hereditary lesions, medications, toxic agents, infections, and medications. Nephrotic syndrome- caused by disorders such as chronic glomerulonephritis, systemic lupus erythematosus, multiple myeloma, and renal vein thrombosis.

The nurse is reviewing the potassium level of a patient with kidney disease. The results of the test are 6.5 mEq/L, and the nurse observes peaked T waves on the ECG. What priority intervention does the nurse anticipate the physician will order to reduce the potassium level?

Administration of sodium polystyrene sulfonate [Kayexalate])

The nurse is caring for a patient who underwent a kidney transplant. The nurse understands that rejection of a transplanted kidney within 24 hours after transplant is termed which of the following?

After a kidney transplant, rejection and failure can occur within 24 hours (hyperacute), within 3 to 14 days (acute), or after many years. A hyperacute rejection is caused by an immediate antibody-mediated reaction that leads to generalized glomerular capillary thrombosis and necrosis.

With prostatitis, what foods should be avoided until the inflammation has resolved?

Alcohol Caffeine

The nurse is assessing a child diagnosed with nephritic syndrome and observes generalized edema. The nurse documents this as which of the following?

Anasarca refers to generalized edema. Enuresis refers to continued incontinence of urine past the age of toilet training. Hydronephrosis refers to a condition in which the pelvis and calyces of the kidney are dilated. Phimosis refers to a condition in which the foreskin of the penis cannot be retracted.

Which of the following stimuli is known to trigger an episode of autonomic dysreflexia in the patient who has suffered a spinal cord injury? a) Diarrhea b) Applying a blanket over the patient c) Placing the patient in a sitting position d) Voiding

Applying a blanket over the patient Explanation: An object on the skin or skin pressure may precipitate an autonomic dysreflexic episode. In general, constipation or fecal impaction triggers autonomic dysreflexia. When the patient is observed to be demonstrating signs of autonomic dysreflexia, he is placed in a sitting position immediately to lower blood pressure. The most common cause of autonomic dysreflexia is a distended bladder

A female client undergoes dialysis as a part of treatment for kidney failure. The client is administered heparin during dialysis, to achieve therapeutic levels. Which of the following steps should the nurse take to allow heparin to be metabolized and excreted in the client?

Avoid administering injections for two to four hours after heparin administration

Which of the following terms is used to describe the concentration of urea and other nitrogenous wastes in the blood? a) Azotemia b) Hematuria c) Proteinuria d) Uremia

Azotemia Explanation: Azotemia is the concentration of urea and other nitrogenous wastes in the blood. Uremia is an excess of urea and other nitrogenous wastes in the blood. Hematuria is blood in the urine. Proteinuria is protein in the urine.

A client with a history of chronic renal failure receives hemodialysis treatments three times per week through an arteriovenous (AV) fistula in the left arm. Which intervention should the nurse include in the care plan? A. Keep the AV fistula wrapped in gauze. B. Assess the AV fistula for a bruit and thrill. C. Take the client's blood pressure in the left arm. D. Keep the AV fistula site dry.

B. Assess the AV fistula for a bruit and thrill.

A client with chronic renal failure (CRF) is admitted to the urology unit. Which diagnostic test results are consistent with CRF? A. Increased serum levels of potassium, magnesium, and calcium B. Blood urea nitrogen (BUN) 100 mg/dL and serum creatinine 6.5 mg/dL C. Increased pH with decreased hydrogen ions D. Uric acid analysis 3.5 mg/dL and phenolsulfonphthalein (PSP) excretion 75%

B. Blood urea nitrogen (BUN) 100 mg/dL and serum creatinine 6.5 mg/dL

The nurse is educating a client who is required to restrict potassium intake. What foods would the nurse suggest the client eliminate that are rich in potassium? A. Butter B. Citrus fruits C. Salad oils D. Cooked white rice

B. Citrus fruits

A client with renal failure is undergoing continuous ambulatory peritoneal dialysis. Which nursing diagnosis is the most appropriate for this client? A. Activity intolerance B. Risk for infection C. Toileting self-care deficit D. Impaired urinary elimination

B. Risk for infection

The presence of prerenal azotemia is a probable indicator for hospitalization for CAP. Which of the following is an initial laboratory result that would alert a nurse to this condition? a) Blood urea nitrogen (BUN)-to-creatinine ratio (BUN:Cr) >20. b) Glomerular filtration rate (GFR) of 100 mL/min. c) BUN of 18 mg/dL. d) Serum creatinine of 1.2 mg/dL.

Blood urea nitrogen (BUN)-to-creatinine ratio (BUN:Cr) >20. Explanation: The normal BUN:Cr ratio is less than 15. Prerenal azotemia is caused by hypoperfusion of the kidneys due to a nonrenal cause. Over time, higher than normal blood levels of urea or other nitrogen-containing compounds will develop.

Which assessment finding is most important in determining nursing care for a client with acute glomerulonephritis? a) Blurred vision b) Peripheral edema c) Dark smoky colored urine d) Presence of albumin in the urine

Blurred vision Explanation: Visual disturbances can be indicative of rising blood pressure in a client with acute glomerulonephritis. Severe hypertension needs prompt treatment to prevent convulsions. Presence of albumin (protein) and RBCs in the urine, along with periorbital and peripheral edema, are common symptoms associated with glomerulonephritis.

Which assessment finding is most important in determining nursing care for a client with acute glomerulonephritis?

Blurred vision. Visual disturbances can be indicative of rising blood pressure in a client with acute glomerulonephritis.

What is a treatment for prostate cancer most commonly used other than surgical removal?

Brachytherapy

A physician orders regular insulin 10 units I.V. along with 50 ml of dextrose 50% for a client with acute renal failure. What electrolyte imbalance is this client most likely experiencing? a) Hyperglycemia b) Hypercalcemia c) Hyperkalemia d) Hypernatremia

C) Hyperkalemia Administering regular insulin I.V. concomitantly with 50 ml of dextrose 50% helps shift potassium from the extracellular fluid into the cell, which normalizes serum potassium levels in the client with hyperkalemia. This combination doesn't help reverse the effects of hypercalcemia, hypernatremia, or hyperglycemia.

When preparing a client for hemodialysis, which of the following would be most important for the nurse to do?

Check for thrill or bruit over the access site.

A 24-year-old patient was admitted to the emergency room after a water skiing accident. The X-rays revealed two fractured vertebrae, T-12 and L1. Based on this information, the nurse would know to perform which of the following actions?

Check the patient's urine for hematuria.

Diet modifications are part of nutritional therapy for the management of ARF. What high-potassium food should be restricted.

Citrus fruits

The nurse is educating a client who is required to restrict potassium intake. What foods would the nurse suggest the client eliminate that are rich in potassium?

Citrus fruits

When obtaining a urine specimen from a school age child, what method is preferred?

Clean catch specimen

The client is admitted to the hospital with a diagnosis of acute glomerulonephritis. Which clinical manifestation would the nurse expect to find? a) Hypotension b) Hyperalbuminemia c) Peripheral neuropathy d) Cola-colored urine

Cola-colored urine Explanation: Clinical manifestations of acute glomerulonephritis include cola-colored urine, hematuria, edema, azotemia, and proteinuria.

Which type of brain injury has occurred if the patient may be aroused with effort, but soon slips back into unconsciousness? a) Intracranial hemorrhage b) Diffuse axonal injury c) Contusion d) Concussion

Contusion Explanation: A patient with a contusion may be aroused with effort but soon slips back into unconsciousness. A concussion is a temporary loss of neurologic function with no apparent structural damage. A diffuse axonal injury involves widespread damage to the axons in the cerebral hemispheres, corpus callosum, and brain stem. An intracranial hemorrhage is a collection of blood that develops within the cranial vault.

A client develops decreased renal function and requires a change in antibiotic dosage. On which factor should the physician base the dosage change? a) Therapeutic index b) GI absorption rate c) Liver function studies d) Creatinine clearance

D) Creatinine clearance The physician should base changes to antibiotic dosages on creatinine clearance test results, which gauge the kidney's glomerular filtration rate; this factor is important because most drugs are excreted at least partially by the kidneys. The GI absorption rate, therapeutic index, and liver function studies don't help determine dosage change in a client with decreased renal function.

Which of the following would the nurse expect to find when reviewing the laboratory test results of a client with renal failure? a) Increased red blood cell count b) Decreased serum potassium level c) Increased serum calcium level d) Increased serum creatinine level

D) Increased serum creatinine level Explanation: In renal failure, laboratory blood tests reveal elevations in BUN, creatinine, potassium, magnesium, and phosphorus. Calcium levels are low. The RBC count, hematocrit, and hemoglobin are decreased.

A client has end-stage renal failure. Which of the following should the nurse include when teaching the client about nutrition to limit the effects of azotemia? A. Increase protein, carbohydrates, and fat intake. B. Eliminate fat intake and increase protein intake. C. Increase fat intake and limit carbohydrates. D. Increase carbohydrates and limit protein intake.

D. Increase carbohydrates and limit protein intake.

Hyperkalemia is a serious side effect of acute renal failure. Identify the electrocardiogram (ECG) tracing that is diagnostic for hyperkalemia. A. Prolonged ST segment B. Multiple spiked P waves C. Shortened QRS complex D. Tall, peaked T waves

D. Tall, peaked T waves

A client who is blind is admitted for treatment of gastroenteritis. Which nursing diagnosis takes highest priority for this client? a) Impaired physical mobility b) Deficient fluid volume c) Activity intolerance d) Risk for injury

Deficient fluid volume Explanation: Because the client has gastroenteritis and is probably dehydrated, Deficient fluid volume takes highest priority. A sensory deficit such as blindness puts the client at risk for injury from the environment; however, a potential problem doesn't take highest priority. Although Activity intolerance or Impaired physical mobility also may be relevant, these nursing diagnoses don't take precedence over the client's dehydration.

A nurse identifies a nursing diagnosis of risk for ineffective breathing pattern related to incisional pain and restricted positioning for a client who has had a nephrectomy. Which of the following would be most appropriate for the nurse to include in the client's plan of care?

Encourage use of incentive spirometer every 2 hours.

During hemodialysis, excess water is removed from the blood by which of the following?

Excess water is removed from the blood by osmosis, in which water moves from an area of higher solute concentration in the blood toward an area of lower solute concentration into the dialysate

Which nursing assessment finding indicates that the client who has undergone renal transplant has not met expected outcomes?

FEVER

An infant is diagnosed with a urinary tract infection. What would the nurse expect on assessment?

Failure to thrive

Which of the following causes should the nurse suspect in a client is diagnosed with intrarenal failure?

Glomerulonephritis

What s/s could point to renal or bladder cancer?

Hematuria

Twenty-four hours after undergoing kidney transplantation, a client develops a hyperacute rejection. To correct this problem, the nurse should prepare the client for:

Hyperacute rejection isn't treatable; the only way to stop this reaction is to remove the transplanted organ or tissue.

As renal failure progresses and the glomerular filtration rate (GFR) falls, which of the following changes occur?

Hyperphosphatemia

The nurse is caring for a child admitted with a urinary tract infection. In addition to foul smelling urine, which of the following clinical manifestations would likely have been noted in the child with this diagnosis?

In children, the symptoms or a urinary tract infection may be fever, nausea, vomiting, foul-smelling urine, weight loss, and increased urination. Occasionally there is little or no fever. Vomiting is common, and diarrhea may occur.

While presenting a panel discussion to a group of parents about urinary tract infections (UTIs) in children, one of the parents asks the nurse, "Why would my daughter be more at risk than my son?" Which response by the nurse would be most accurate?

In females, the urethra is shorter, which allows bacteria to enter the bladder. It also is closer in physical proximity to the rectum, leading to possible contamination.

A client has end-stage renal failure. Which of the following should the nurse include when teaching the client about nutrition to limit the effects of azotemia?

Increase carbohydrates and limit protein intake.

Which of the following causes should the nurse suspect in a client is diagnosed with intrarenal failure?

Intrarenal causes of renal failure include prolonged renal ischemia, nephrotoxic agents, and infectious processes such as acute glomerulonephritis.

A client with acute renal failure progresses through four phases. Which describes the onset phase?

It is accompanied by reduced blood flow to the nephrons.

The client with acute renal failure progresses through four phases. Which of the following describes the initiation phase? a) Fluid volume excess develops, which leads to edema, hypertension, and cardiopulmonary complications. b) It is accompanied by reduced blood flow to the nephrons. c) Normal glomerular filtration and tubular function are restored. d) The excretion of wastes and electrolytes continues to be impaired despite increased water content of the urine.

It is accompanied by reduced blood flow to the nephrons. Explanation: The initiation phase is accompanied by reduced blood flow to the nephrons. In the oliguric phase, fluid volume excess develops, which leads to edema, hypertension, and cardiopulmonary complications. During the diuretic phase, excretion of wastes and electrolytes continues to be impaired despite increased water content of the urine. During the recovery phase, normal glomerular filtration and tubular function are restored.

Which of the following is used to decrease potassium level seen in acute renal failure?

Kayexalate Kayexalate works by exchanging sodium ions for potassium ions in the intestinal tract

For a client in the oliguric phase of acute renal failure (ARF), which nursing intervention is the most important? a) Encouraging coughing and deep breathing b) Promoting carbohydrate intake c) Limiting fluid intake d) Providing pain-relief measures

Limiting fluid intake Explanation: During the oliguric phase of ARF, urine output decreases markedly, possibly leading to fluid overload. Limiting oral and I.V. fluid intake can prevent fluid overload and its complications, such as heart failure and pulmonary edema. Encouraging coughing and deep breathing is important for clients with various respiratory disorders. Promoting carbohydrate intake may be helpful in ARF but doesn't take precedence over fluid limitation. Controlling pain isn't important because ARF rarely causes pain.

Examination of a client's bladder stones reveal that they are primarily composed of uric acid. The nurse would expect to provide the client with which type of diet?

Low purine

A nurse is performing postoperative care on a child with a ureteral stent. Which of the following interventions will help manage tube patency?

Monitor output.

A client is scheduled for a renal angiography. Which of the following would be appropriate before the test?

Monitor the client for an allergy to iodine contrast material.

Treatment of metabolic acidosis in chronic renal failure includes: a) Hemodialysis b) Sodium bicarbonate supplements c) Peritoneal dialysis d) No treatment

No treatment Explanation: The metabolic acidosis of chronic renal failure usually produces no symptoms and requires no treatment.

The nurse cares for a client with acute kidney injury (AKI). The client is experiencing an increase in the serum concentration of urea and creatinine. The nurse determines the client is experiencing which phase of AKI?

Oliguria R: The oliguria period is accompanied by an increase in the serum concentration of substances usually excreted by the kidneys (urea, creatinine, uric acid, organic acids, and the intracellular cations [potassium and magnesium])

The nurse is teaching a group of nursing students about acute glomerulonephritis genitourinary conditions. A student asks the about a condition that occurs when there is a decreased volume of urine output. The condition the student is referring to is which of the following?

Oliguria is a subnormal volume of urine.

An investment banker, with chronic renal failure, informs the nurse of the choice for continuous cyclic peritoneal dialysis. Which is the best response by the nurse?

Once a treatment choice has been selected by the client, the nurse should support the client in that decision. Continuous cyclic peritoneal dialysis will provide more independence for this client and supports the client's decision for treatment mode. The risk of peritonitis is greater, and symptoms should be discussed as part of the management of the disorder. Peritoneal dialysis is an effective method of dialysis for many clients

A nurse who provides care in a high-acuity medical setting is aware of the high incidence and morbidity of acute renal failure (ARF). To reduce patients' risks of developing ARF during their stay in hospital, it is imperative that:

Patients' medication regimens be monitored closely

When assessing the impact of medications on the etiology of acute renal failure, the nurse recognizes which of the following as the drug that is not nephrotoxic?

Penicillin

Which assessment finding is common in children diagnosed with nephrotic syndrome?

Periorbital edema

BPH meds

Proscar- liver testing, takes awhile to work, must not be handled by women or children Flomax- increase urine flow in BPH, can cause ortho-hypotension Surgery- TURP

What type of dietary restrictions should a patient with acute glomerulonephritis have?

Protein

The hallmark sign of nephrotic syndrome is...

Proteinuria

A client with renal failure is undergoing continuous ambulatory peritoneal dialysis. Which nursing diagnosis is the most appropriate for this client?

Risk for infection

The nurse understands that common causes of acute glomerulonephritis in pediatric patients include:

Scarlet fever

With any process involving scrotal swelling, what nursing intervention can promote comfort?

Scrotal elevation

Based on her knowledge of the primary cause of ESRD, the nurse knows to assess the most important indicator. What is that indicator? a) Serum glucose b) Urine protein c) Blood pressure d) pH and HCO3

Serum glucose Explanation: The nurse would evaluate serum and urine levels of glucose because diabetes is the primary cause of renal failure.

A teacher sends a child to see the school nurse for irritability and bruising. Which of the following symptoms would be indicative of hemolytic uremic syndrome?

Signs of hemolytic uremic syndrome include oliguria, irritability, jaundice, bloody diarrhea, purpura, ecchymosis, and pallor 5 to 10 days after a prodromal illness. The child also usually experiences anorexia, slight fevers, and can become lethargic. Symptoms of polyuria, weight gain, high fever, and dysuria are not typically seen with hemolytic uremic syndrome.

A client is admitted with nausea, vomiting, and diarrhea. His blood pressure on admission is 74/30 mm Hg. The client is oliguric and his blood urea nitrogen (BUN) and creatinine levels are elevated. The physician will most likely write an order for which treatment?

Start I.V. fluids with a normal saline solution bolus followed by a maintenance dose.

A client is admitted with nausea, vomiting, and diarrhea. His blood pressure on admission is 74/30 mm Hg. The client is oliguric and his blood urea nitrogen (BUN) and creatinine levels are elevated. The physician will most likely write an order for which treatment?

Start I.V. fluids with a normal saline solution bolus followed by a maintenance dose. This treatment should rehydrate the client, causing his blood pressure to rise, his urine output to increase, and the BUN and creatinine levels to normalize.

A client is admitted with nausea, vomiting, and diarrhea. His blood pressure on admission is 74/30 mm Hg. The client is oliguric and his blood urea nitrogen (BUN) and creatinine levels are elevated. The physician will most likely write an order for which treatment?

Start IV fluids with a normal saline solution bolus followed by a maintenance dose. R: in prerenal failure caused by hypovolemia

Autonomic dysreflexia can occur with spinal cord injuries above which of the following levels? a) T10 b) T6 c) S2 d) L4

T6 Explanation: Any patient with a lesion above T6 segment is informed that autonomic dysreflexia can occur and that it may occur even years after the initial injury.

Hyperkalemia is a serious side effect of acute renal failure. Identify the electrocardiogram (ECG) tracing that is diagnostic for hyperkalemia.

Tall peaked T

Hyperkalemia is a serious side effect of acute renal failure. Identify the electrocardiogram (ECG) tracing that is diagnostic for hyperkalemia.

Tall, peaked T waves

When caring for the patient with acute glomerulonephritis, which of the following assessment findings should the nurse anticipate?

Tea-colored urine

When caring for the patient with acute glomerulonephritis, which of the following assessment findings should the nurse anticipate? a) Tea-colored urine b) Low blood pressure c) Left upper quadrant pain d) Pyuria

Tea-colored urine Explanation: Tea-colored urine is a typical symptom of glomerulonephritis. Flank pain on the affected side, not left upper quadrant pain, would be present. Pyuria is a symptom of pyelonephritis, not glomerulonephritis. Blood pressure typically elevates in glomerulonephritis.

What is the most common type of cancer among men age 15-35? END PP QUESTIONS

Testicular

A nurse is providing postoperative care for a 5-year-old who had a hypospadias repair. Which of the following would the nurse expect for postoperative care of this child?

The child is usually hospitalized for under 24 hours with a hypospadias repair. Wheelchair kickball is an appropriate sport since typically the child is not in much pain after the surgery. This will help increase gross motor activity. The tubing should remain unclamped and free from obstruction at all times.

An expected outcome for the hemodialysis client is:

The client explains how to assess the venous access site.

After teaching a group of students about how to perform peritoneal dialysis, which statement would indicate to the instructor that the students need additional teaching?

The dialysate should be warmed in a commercial warmer and never in a microwave oven. Strict aseptic technique is essential. The infusion clamp is opened during the infusion and clamped after the infusion. When the dwell time is done, the drain clamp is opened and the fluid is allowed to drain by gravity into the drainage bag.

The nurse performs acute intermittent peritoneal dialysis (PD) on a client who is experiencing uremic signs and symptoms. The peritoneal fluid is not draining as expected. What is the best response by the nurse?

Turn the client from side to side.

Which clinical finding should a nurse look for in a client with chronic renal failure?

Uremia is the buildup of nitrogenous wastes in the blood, evidenced by an elevated blood urea nitrogen and creatine levels. Uremia, anemia, and acidosis are consistent clinical manifestations of chronic renal failure.

A group of students is reviewing for a test on the urinary and renal system. The students demonstrate understanding of the information when they identify which of the following as part of the upper urinary tract?

Ureters

A client comes to the Emergency Department complaining of a sudden onset of sharp, severe flank pain. During the physical examination, the client indicates that the pain, which comes in waves, travels to the suprapubic region. He states, "I can even feel the pain at the tip of my penis." Which of the following would the nurse suspect?

Urinary calculi

Which of the following is the most common site of a nosocomial infection?

Urinary tract

A nurse is caring for a client diagnosed with acute renal failure. The nurse notes on the intake and output record that the total urine output for the previous 24 hours was 35 ml. Urine output that's less than 50 ml in 24 hours is known as:

Urine output less than 50 ml in 24 hours is called anuria. Urine output of less than 400 ml in 24 hours is called oliguria. Polyuria is excessive urination. Hematuria is the presence of blood in the urine.

A change that occurs during chronic glomerulonephritis is termed

anemia

A client admitted with a gunshot wound to the abdomen is transferred to the intensive care unit after an exploratory laparotomy. I.V. fluid is being infused at 150 ml/hour. Which assessment finding suggests that the client is experiencing acute renal failure (ARF)?

Urine output of 250 ml/24 hours

A client with a history of chronic renal failure receives hemodialysis treatments three times per week through an arteriovenous (AV) fistula in the left arm. Which intervention should the nurse include in the care plan?

assess the AV fistula for a bruit and thrill because if these findings aren't present, the fistula isn't functioning. The AV fistula may get wet when the client isn't being dialyzed. Immediately after a dialysis treatment, the access site should be covered with adhesive bandages, not gauze. Blood pressure readings or venipunctures shouldn't be taken in the arm with the AV fistula.

Which nursing assessment finding indicates that the client who has undergone renal transplant has not met expected outcomes? a) Weight loss b) Fever c) Absence of pain d) Diuresis

b) Fever Fever is an indicator of infection or transplant rejection.

The client with chronic renal failure complains of intense itching. Which assessment finding would indicate the need for further nursing education?

breif hot showers

The client with chronic renal failure is exhibiting signs of anemia. Which is the best nursing rationale for this symptom?

diminished erythropoietin production

A client with chronic renal failure (CRF) has developed faulty red blood cell (RBC) production. The nurse should monitor this client for:

fatigue and weakness

The nurse is assessing a child with acute poststreptococcal glomerulonephritis. Which of the following would the nurse expect to assess? Select all answers that apply.

fatigue, lethargy, abdominal pain, hypertension, crackles, and anorexia.

Which nursing assessment finding indicates that the client who has undergone renal transplant has not met expected outcomes?

fever

What are the primary presenting features of acute glomerulonephritis

hematuria edema azotemia proteinuria (>3 to 5 g/day)

Which of the following is a characteristic of the intrarenal category of acute renal failure?

increased BUN, increased creatinine, a low specific gravity of urine, and increased urine sodium.

A client is having a blood urea nitrogen (BUN) test. BUN level is:

increased in renal disease and urinary obstruction.

After teaching a group of students about how to perform peritoneal dialysis, which statement would indicate to the instructor that the students need additional teaching?

its approp to warm in the dialstat in micro wave

Which of the following is used to decrease potassium level seen in acute renal failure?

kayexalate

What is a risk factor for renal cancer?

obesity

The nurse is caring for a patient with ESKD. Which of the following acid-base imbalances is associated with this disorder? a) pH 7.31, PaCO2 48, HCO3 24- b) pH 7.47, PaCO2 45, HCO3 33- c) pH 7.20, PaCO2 36, HCO3 14- d) pH 7.50, PaCO2 29, HCO3 22-

pH 7.20, PaCO2 36, HCO3 14- Explanation: Metabolic acidosis occurs in ESKD because the kidneys are unable to excrete increased loads of acid. Decreased acid secretion results from the inability of the kidney tubules to excrete ammonia (NH3-) and to reabsorb sodium bicarbonate (HCO3-). There is also decreased excretion of phosphates and other organic acids.

The nurse is caring for a patient with ESKD. Which of the following acid-base imbalances is associated with this disorder?

pH 7.20, PaCO2 36, HCO3 14-. Metabolic acidosis occurs in ESKD because the kidneys are unable to excrete increased loads of acid.

A client has pyelonephritis and is undergoing parenteral antibiotic treatment. What will be the effect of the infection on the client's kidneys?

renal scarring

A client presents to the ED reporting left flank pain and lower abdominal pain. The pain is severe, sharp, stabbing, and colicky in nature. The client has also experienced nausea and emesis. The nurse suspects the client is experiencing:

ureteral stones.

The most accurate indicator of fluid loss or gain in an acutely ill patient

weight

The nurse is caring for a patient with a medical history of untreated CKD that has progressed to ESKD. Which of the following serum values and associated signs and symptoms will the nurse expect the patient to exhibit? Select all that apply.

• Calcium 7.5 mg/dL; hypotension and irritability • Potassium 6.4 mEq/L; dysrhythmias and abdominal distention • Phosphate 5.0 mg/dL; tachycardia and nausea and emesis

Compliance to a renal diet is a difficult lifestyle change for a patient on hemodialysis. The nurse should reinforce nutritional information. Which of the following teaching points should be included? Select all that apply. a) Restrict sodium to 2,000 to 3,000 mg daily. b) Restrict fluid to daily urinary output plus 500 to 800 mL. c) Limit protein to 1.6 g/kg/day. d) Eat foods such as milk, fish, and eggs. e) Increase potassium to prevent cardiac problems.

• Eat foods such as milk, fish, and eggs. • Restrict sodium to 2,000 to 3,000 mg daily. • Restrict fluid to daily urinary output plus 500 to 800 mL. Explanation: With hemodialysis, protein should be limited to 1.2 to 1.3 g/kg/24 hr. Potassium, along with sodium and phosphorus should be restricted.

The nurse is providing supportive care to a client receiving hemodialysis in the management of acute renal failure. Which statement from the nurse best reflects the ability of the kidneys to recover from acute renal failure? A. Acute renal failure tends to turn to end-stage failure. B. The kidneys can improve over a period of months. C. Kidney function will improve with transplant. D. Once on dialysis, the need will be permanent.

B. The kidneys can improve over a period of months.

A nurse assesses a client shortly after living donor kidney transplant surgery. Which postoperative finding must the nurse report to the physician immediately? A. Serum potassium level of 4.9 mEq/L B. Urine output of 20 ml/hour

B. Urine output of 20 ml/hour

The nurse is providing care to a client who has had a kidney biopsy. The nurse would need to be alert for signs and symptoms of which of the following?

Bleeding

A client with chronic renal failure (CRF) is admitted to the urology unit. Which diagnostic test results are consistent with CRF?

Blood urea nitrogen (BUN) 100 mg/dl and serum creatinine 6.5 mg/dl. normal BUN level ranges 8 to 23 mg/dl; the normal serum creatinine level ranges from 0.7 to 1.5 mg/dl.

A client is in end-stage chronic renal failure and is being added to the transplant list. The nurse explains to the client how donors are found for clients needing kidneys. Which statement is accurate?

Donors are selected from compatible living donors.

The clinic nurse is teaching a young woman about preventing recurrent urinary tract infections. What information should the nurse include?

Drink liberal amounts of fluids.

The first method of choice for obtaining a urine specimen from a 3-year-old child with a possible urinary tract infection is which of the following?

In the cooperative, toilet-trained child, a clean midstream urine may be used successfully to obtain a "clean catch" voided urine. If a culture is needed, the child may be catheterized, but this is usually avoided if possible. A suprapubic aspiration also may be done to obtain a sterile specimen.

What is a characteristic of the intrarenal category of acute kidney injury (AKI)?

Increased BUN

What is a characteristic of the intrarenal category of acute renal failure?

Increased BUN

The nurse reviews a client's history and notes that the client has a history of hyperparathyroidism. The nurse would identify that this client most likely would be at risk for which of the following?

Kidney stones

The nurse is providing an education program for the nursing assistants in a long-term care facility in order to decrease the number of UTIs in the female population. What interventions should the nurse introduce in the program? Select all that apply.

Perform hand hygiene prior to patient care. Assist the patients with frequent toileting. Provide careful perineal care.

Which of the following occurs late in chronic glomerulonephritis?

Peripheral neuropathy with diminished deep tendon reflexes and neurosensory changes occur late in the disease. The patient becomes confused and demonstrates a limited attention span. An additional late finding includes evidence of pericarditis with or without a pericardial friction rub. The first indication of disease may be a sudden, severe nosebleed, a stroke, or a seizure.

A 5-year-old child with acute renal failure develops hyperkalemia. Which of the following would the nurse expect to administer?

Polystyrene sulfonate (Kayexalate) is used to decrease potassium levels.

What is a hallmark of the diagnosis of nephrotic syndrome?

Proteinuria

Which term refers to inflammation of the renal pelvis?

Pyelonephritis

A patient has been diagnosed with postrenal failure. The nurse reviews the patient's electronic health record and notes a possible cause. Which of the following is the possible cause?

Renal Calculi R: Postrenal ARF is the result of an obstruction that develops anywhere from the collecting ducts of the kidney to the urethra. This results from ureteral blockage, such as from bilateral renal calculi or benign prostatic hypertrophy (BPH).

The laboratory results for a patient with renal failure, accompanied by decreased glomerular filtration, would be evaluated frequently. Which of the following is the most sensitive indicator of renal function? a) Creatinine clearance of 90 mL/min b) Serum creatinine of 1.5 mg/dL c) BUN of 20 mg/dLb d) Urinary protein level of 150 mg/24h.

Serum creatinine of 1.5 mg/dL Explanation: As glomerular filtration decreases, the serum creatinine and BUN levels increase and the creatinine clearance decreases. Serum creatinine is the more sensitive indicator of renal function because of its constant production in the body. The BUN is affected not only by renal disease but also by protein intake in the diet, tissue catabolism, fluid intake, parenteral nutrition, and medications such as corticosteroids.

A client admitted with a gunshot wound to the abdomen is transferred to the intensive care unit after an exploratory laparotomy. IV fluid is being infused at 150 mL/hour. Which assessment finding suggests that the client is experiencing acute renal failure (ARF)?

Urine output of 250 ml/24 hours

Twenty-four hours after undergoing kidney transplantation, a client develops a hyperacute rejection. To correct this problem, the nurse should prepare the client for: a) intra-abdominal instillation of methylprednisolone sodium succinate (Solu-Medrol). b) high-dose I.V. cyclosporine (Sandimmune) therapy. c) removal of the transplanted kidney. d) bone marrow transplant.

removal of the transplanted kidney. Explanation: Hyperacute rejection isn't treatable; the only way to stop this reaction is to remove the transplanted organ or tissue. Although cyclosporine is used to treat acute transplant rejection, it doesn't halt hyperacute rejection. Bone marrow transplant isn't effective against hyperacute rejection of a kidney transplant. Methylprednisolone sodium succinate may be given I.V. to treat acute organ rejection, but it's ineffective against hyperacute rejection

A client with renal failure is undergoing continuous ambulatory peritoneal dialysis. Which nursing diagnosis is the most appropriate for this client?

risk for infection

A client diagnosed with acute kidney injury (AKI) has a serum potassium level of 6.5 mEq/L. The nurse anticipates administering:

sodium polystyrene sulfonate (Kayexalate)

Which of the following symptoms are indicative of a rapidly expanding acute subdural hematoma? Select all that apply. a) Coma b) Tachypnea c) Decreased reactivity of the pupils d) Hemiparesis e) Hypotension f) Bradycardia

• Hemiparesis • Decreased reactivity of the pupils • Bradycardia • Coma Explanation: Signs and symptoms include changes in the level of consciousness (LOC), changes in the reactivity of the pupils, and hemiparesis (weakness on one side of the body). There may be minor or even no symptoms, with small collections of blood. Coma, increasing blood pressure, decreasing heart rate, and slowing respiratory rate are all signs of a rapidly expanding mass requiring immediate intervention

Mr. Jarvis's renal failure has become chronic. You are seeing him in clinic and he discusses the various signs and symptoms he is experiencing. Select all of the following which you know to be associated with chronic renal failure.

• Lethargy • Muscle cramps • Bleeding of the oral mucous membranes

A client who has been treated for chronic renal failure (CRF) is ready for discharge. The nurse should reinforce which dietary instruction? a) "Drink plenty of fluids, and use a salt substitute." b) "Eat plenty of bananas." c) "Increase your carbohydrate intake." d) "Be sure to eat meat at every meal."

"Increase your carbohydrate intake." Explanation: A client with CRF requires extra carbohydrates to prevent protein catabolism. In a client with CRF, unrestricted intake of sodium, protein, potassium, and fluid may lead to a dangerous accumulation of electrolytes and protein metabolic products, such as amino acids and ammonia. Therefore, the client must limit intake of sodium; meat, which is high in protein; bananas, which are high in potassium; and fluid, because the failing kidneys can't secrete adequate urine. Salt substitutes are high in potassium and should be avoided.

Which statement by the client with end-stage renal disease indicates teaching by the nurse was effective?

"Ultrafiltration methods take much longer than hemodialysis.".Ultrafiltration methods (CVVH, CVVHD) are better tolerated by unstable clients as fluid is removed slowly, resulting in mild hemodynamic effects.

What is true about extracorporeal shock wave lithotripsy (ESWL)? Select all that apply.

-Stones are shattered into smaller particles that are passed from the urinary tract. -ESWL is administered with the client in a water bath or surrounded by a soft cushion while under light anesthesia or sedation.

When preparing a client for hemodialysis, which of the following would be most important for the nurse to do? a) Check for thrill or bruit over the access site. b) Warm the solution to body temperature. c) Inspect the catheter insertion site for infection. d) Add the prescribed drug to the dialysate.

A) Check for thrill or bruit over the access site. When preparing a client for hemodialysis, the nurse would need to check for a thrill or bruit over the vascular access site to ensure patency. Inspecting the catheter insertion site for infection, adding the prescribed drug to the dialysate, and warming the solution to body temperature would be necessary when preparing a client for peritoneal dialysis.

A client is admitted with nausea, vomiting, and diarrhea. His blood pressure on admission is 74/30 mm Hg. The client is oliguric and his blood urea nitrogen (BUN) and creatinine levels are elevated. The physician will most likely write an order for which treatment? a) Start I.V. fluids with a normal saline solution bolus followed by a maintenance dose. b) Administer furosemide (Lasix) 20 mg I.V. c) Encourage oral fluids. d) Start hemodialysis after a temporary access is obtained.

A) Start IV fluids with normal saline solution bolus followed by a maintenance dose. Explanation: The client is in prerenal failure caused by hypovolemia. I.V. fluids should be given with a bolus of normal saline solution followed by maintenance I.V. therapy. This treatment should rehydrate the client, causing his blood pressure to rise, his urine output to increase, and the BUN and creatinine levels to normalize. The client wouldn't be able to tolerate oral fluids because of the nausea, vomiting, and diarrhea. The client isn't fluid-overloaded so his urine output won't increase with furosemide, which would actually worsen the client's condition. The client doesn't require dialysis because the oliguria and elevated BUN and creatinine levels are caused by dehydration.

A client, aged 75, is diagnosed with a renal disease and administered nephrotoxic drugs in normal doses. The nurse is aware that it is important to observe the client closely for any changes in renal status. Which of the following measures may help a nurse determine a change in renal status? A. Observing the client's urinary output. B. Observing the client's fluid intake. C. Checking for a thrill or a bruit daily. D. Observing the skin color and nail beds.

A. Observing the client's urinary output.

The most accurate indicator of fluid loss or gain in an acutely ill client is: A. weight. B. blood pressure. C. pulse rate. D. edema.

A. weight.

A patient with chronic kidney failure experiences decreased levels of erythropoietin. What serious complication related to those levels should the nurse assess for when caring for this client?

Anemia

what are some neurological manifestations of chronic renal failure

Asterixis (flapping arms) tremors seizures

You are a neuro trauma nurse working in a neuro ICU. What would you know is an acute emergency and is seen in clients with a cervical or high thoracic spinal cord injury after the spinal shock subsides? a) Paraplegia b) Tetraplegia c) Autonomic dysreflexia d) Areflexia

Autonomic dysreflexia Explanation: Autonomic dysreflexia is an acute emergency and is seen in clients with a cervical or high thoracic spinal cord injury, usually after the spinal shock subsides. Tetraplegia results in the paralysis of all extremities when there is a high cervical spine injury. Paraplegia occurs with injuries at the thoracic level. Areflexia is a loss of sympathetic reflex activity below the level of injury within 30 to 60 minutes of a spinal injury.

What is the term for a concentration of urea and other nitrogenous wastes in the blood?

Azotemia

A nurse identifies a nursing diagnosis of risk for ineffective breathing pattern related to incisional pain and restricted positioning for a client who has had a nephrectomy. Which of the following would be most appropriate for the nurse to include in the client's plan of care? A. Keep the drainage catheter below the level of insertion. B. Encourage use of incentive spirometer every 2 hours. C. Administer isotonic fluid therapy as ordered. D. Monitor temperature every 4 hours.

B. Encourage use of incentive spirometer every 2 hours.

A client with renal failure is undergoing continuous ambulatory peritoneal dialysis. Which nursing diagnosis is the most appropriate for this client? a) Impaired urinary elimination b) Toileting self-care deficit c) Risk for infection d) Activity intolerance

C) Risk for infection Explanation: The peritoneal dialysis catheter and regular exchanges of the dialysis bag provide a direct portal for bacteria to enter the body. If the client experiences repeated peritoneal infections, continuous ambulatory peritoneal dialysis may no longer be effective in clearing waste products. Impaired urinary elimination, Toileting self-care deficit, and Activity intolerance may be pertinent but are secondary to the risk of infection.

Patients diagnosed with hypervolemia should avoid sweet or dry food because: a) It obstructs water elimination. b) It can cause dehydration. c) It can lead to weight gain. d) It increases the client's desire to consume fluid.

D) It increases the client's desire to consume fluid The management goal in hypervolemia is to reduce fluid volume. For this reason, fluid is rationed, and the client is advised to take limited amount of fluid when thirsty. Sweet or dry food can increase the client's desire to consume fluid. Sweet or dry food does not obstruct water elimination nor does it cause dehydration. Weight regulation is not part of hypervolemia management except to the extent that it is achieved on account of fluid reduction.

After teaching a group of students about how to perform peritoneal dialysis, which statement would indicate to the instructor that the students need additional teaching? a) "The effluent should be allowed to drain by gravity." b) "It is important to use strict aseptic technique." c) "The infusion clamp should be open during infusion." d) "It is appropriate to warm the dialysate in a microwave."

D) It is appropriate to warm the dialysate in a microwave Explanation: The dialysate should be warmed in a commercial warmer and never in a microwave oven. Strict aseptic technique is essential. The infusion clamp is opened during the infusion and clamped after the infusion. When the dwell time is done, the drain clamp is opened and the fluid is allowed to drain by gravity into the drainage bag.

A nurse assesses a client shortly after living donor kidney transplant surgery. Which postoperative finding must the nurse report to the physician immediately? a) Serum sodium level of 135 mEq/L b) Serum potassium level of 4.9 mEq/L c) Temperature of 99.2° F (37.3° C) d) Urine output of 20 ml/hour

D) Urine output of 20 ml/hour Explanation: Because kidney transplantation carries the risk of transplant rejection, infection, and other serious complications, the nurse should monitor the client's urinary function closely. A decrease from the normal urine output of 30 ml/hour is significant and warrants immediate physician notification. A serum potassium level of 4.9 mEq/L, a serum sodium level of 135 mEq/L, and a temperature of 99.2° F are normal assessment findings.

A client is admitted with nausea, vomiting, and diarrhea. His blood pressure on admission is 74/30 mm Hg. The client is oliguric and his blood urea nitrogen (BUN) and creatinine levels are elevated. The physician will most likely write an order for which treatment? A. Administer furosemide (Lasix) 20 mg IV B. Encourage oral fluids. C. Start hemodialysis after a temporary access is obtained. D. Start IV fluids with a normal saline solution bolus followed by a maintenance dose.

D. Start IV fluids with a normal saline solution bolus followed by a maintenance dose.

A male client who is admitted with the diagnosis of urinary calculi complains of excruciating pain. The pain is suspected to be caused by increased pressure in the renal pelvis. Which measure would be most appropriate to provide pain relief? a) Encourage frequent ambulation. b) Encourage deep-breathing exercises. c) Restrict the client's sodium intake. d) Encourage the client to void every 2 to 3 hours.

Encourage frequent ambulation. Explanation: When a client with urinary calculi complains of excruciating pain, the client should be encouraged to ambulate. This is because the supine position increases colic, while ambulation relieves it. Also, adequate fluid intake should be suggested to promote the passage of stones and to prevent urinary stasis, or the formation of new stones. The client should be encouraged to void when there is a risk of infection related to urinary stasis. The suggestion for restricting sodium intake is offered to a client with chronic glomerulonephritis, not urinary calculi. The nurse should promote deep-breathing exercises to provide relief to a client recovering from surgery who has an ineffective breathing pattern.

A patient diagnosed with chronic renal failure is receiving continuous peritoneal dialysis (PD). The nurse instructs the patient about which of the following diet plans? a) Low-protein diet b) High-calorie diet c) Low-sodium diet d) High-protein diet

High-protein diet Explanation: Because of protein loss with continuous PD, the patient is instructed to eat a high-protein, nutritious diet. The patient is also encouraged to increase his or her daily fiber intake to help prevent constipation, which can impede the flow of dialysate into or out of the peritoneal cavity. A low-protein diet is required to reduce the production of end products of protein metabolism that the kidneys are unable to excrete. Establishing a diet high in calories and low in protein, sodium, and potassium is essential for patients with acute renal failure

Which of the following is a characteristic of the intrarenal category of acute renal failure? a) Decreased creatinine b) Increased BUN c) High specific gravity d) Decreased urine sodium

Increased BUN Explanation: The intrarenal category of acute renal failure encompasses an increased BUN, increased creatinine, a low specific gravity of urine, and increased urine sodium.

The nurse is helping a client to correctly perform peritoneal dialysis at home. Which suggestions should the nurse provide while educating the client? Select all that apply.

Keep the dialysis supplies in a clean area, away from children and pets. Wash hands before handling the catheter. Wear a mask while handling any dialysate solutions if you have a respiratory infection.

Which period of acute renal failure is accompanied by an increase in the serum concentration of substances usually excreted by the kidneys?

Oliguria

During hemodialysis, excess water is removed from the blood by which of the following?

Osmosis R:Excess water is removed from the blood by osmosis, in which water moves from an area of higher solute concentration in the blood toward an area of lower solute concentration into the dialysate.

A child is getting a diagnostic work-up for nephrotic syndrome. Which of the following lab results would the nurse expect to see?

Proteinuria, hypoalbuminemia, and hypercholesterolemia are diagnostic of a child with nephritic syndrome. The child will also present symptomatically with a sudden onset of edema. Hematuria is typically seen with glomerulonephritis.

With a patient with a foley catheter and a nephrostomy tube, how would you record urine output?

Record foley and nephrostomy separate

The nurse is providing supportive care to a client receiving hemodialysis in the management of acute renal failure. Which statement from the nurse best reflects the ability of the kidneys to recover from acute renal failure?

The kidneys can improve over a period of months. Correct

The client with polycystic kidney disease asks the nurse, "Will my kidneys ever function normally again?" The best response by the nurse is:

There is no cure for polycystic kidney disease. Medical management includes therapies to control blood pressure, urinary tract infections, and pain. Renal replacement therapy is indicated as the kidneys fail.

Which of the following is a strategy to promote urinary continence?

Void regularly, 5 to 8 times a day

The most accurate indicator of fluid loss or gain in an acutely ill patient is a) pulse rate. b) blood pressure. c) edema. d) weight.

Weight. Explanation: The most accurate indicator of fluid loss or gain in an acutely ill patient is weight. An accurate daily weight must be obtained and recorded. Blood pressure, pulse rate, and edema are not the most accurate indicator of fluid loss or gain.

The nurse is performing double diapering for a male infant with hypospadias who has undergone a surgical repair. The nurse performs the following steps. Place the steps in the order in which the nurse performs them.

When performing double diapering, the nurse cuts a hole or a cross-shaped slit in the front of the smaller diaper and then unfolds both diapers, placing the smaller diaper (with the hole) inside the larger one. Next, the nurse places both diapers under the child and carefully brings the penis (if applicable) and catheter/stent through the hole in the smaller diaper, closing the diaper. Finally, the nurse closes the larger diaper, making sure the tip of the catheter/stent is inside the larger diaper.

The nurse is administering calcium acetate (PhosLo) to a patient with end-stage renal disease. When is the best time for the nurse to administer this medication?

With food

A clinical situation in which the increased release of erythropoietin would be expected is: a) hypoexmia b) hypotension c) hyperkalemia d) fluid overload

a) Hypoexmia Erythropoietin is released when the oxygen tension of the renal blood supply is low and stimulates production of red blood cells in the bone marrow. Hypotension causes activation of the renin-angiotensin-aldosterone system, as well as release of ADH. Hyperkalemis stimulates release of aldosterone from the adrenal cortex, and fluid overload does not directly stimulate factors affecting the kidney.

You care for a 3-year-old with hypospadias. After a surgical repair, he has a urethral urinary catheter inserted. You would want to teach his parents that

the catheter insertion site will leave only a minimal scar.

The nurse cares for a client with end-stage kidney disease (ESKD). Which acid-base imbalance is associated with this disorder?

pH 7.20, PaCO2 36, HCO3 14-

Patient education regarding a fistulae or graft includes which of the following? Select all that apply. a) No tight clothing. b) Cleanse site b.i.d. c) No IV or blood pressure taken on extremity with dialysis access. d) Avoid compression of the site. e) Check daily for thrill and bruit.

• Check daily for thrill and bruit. • Avoid compression of the site. • No IV or blood pressure taken on extremity with dialysis access. • No tight clothing. Explanation: The nurse teaches the patient with fistulae or grafts to check daily for a thrill and bruit. Further teaching includes avoiding compression of the site; not permitting blood to be drawn, an IV to be inserted, or blood pressure to be taken on the extremity with the dialysis access; not to wear tight clothing, carry bags or pocketbooks on that side, and not lie on or sleep on the area. The site is not cleansed unless it is being accessed for hemodialysis.

A child is brought into the clinic with symptoms of periorbital edema and dark brown rusty urine. Which nursing assessment finding would best assist in determining the cause of this problem?

sore throat 2 wks ago

A client with chronic renal failure (CRF) is receiving a hemodialysis treatment. After hemodialysis, the nurse knows that the client is most likely to experience: a) increased urine output b) hematuria. c) weight loss. d) increased blood pressure.

weight loss. Explanation: Because CRF causes loss of renal function, the client with this disorder retains fluid. Hemodialysis removes this fluid, causing weight loss. Hematuria is unlikely to follow hemodialysis because the client with CRF usually forms little or no urine. Hemodialysis doesn't increase urine output because it doesn't correct the loss of kidney function, which severely decreases urine production in this disorder. By removing fluids, hemodialysis decreases rather than increases the blood pressure.

Which of the following activities would the patient with a T4 spinal cord injury be able to perform independently? Select all that apply. a) Ambulating b) Eating c) Breathing d) Writing e) Transferring to a wheelchair

• Eating • Breathing • Transferring to a wheelchair • Writing Explanation: Eating, breathing, transferring to a wheelchair, and writing are functional abilities for those with a T4 injury. Ambulation can be performed independently by a patient with a T11-S5 injury.

In a spinal cord injury, neurogenic shock develops due to loss of the autonomic nervous system functioning below the level of the lesion. Which of the following indicators of neurogenic shock would the nurse expect to find? Select all that apply. a) Venous pooling b) Tachypnea c) Hypotension d) Diaphoresis e) Tachycardia f) Hypothermia

• Hypotension • Venous pooling • Tachypnea • Hypothermia Explanation: The vital organs are affected in a spinal cord injury, causing the blood pressure and heart rate to decrease. This loss of sympathetic innervation causes a variety of other clinical manifestations, including a decrease in cardiac output, venous pooling in the extremities, and peripheral vasodilation resulting in mild hypotension, bradycardia, and warm skin. In addition, the patient does not perspire on the paralyzed portions of the body because sympathetic activity is blocked; therefore, close observation is required for early detection of an abrupt onset of fever.

The nurse is caring for a patient diagnosed with chronic glomerulonephritis. The nurse will observe the patient for the development of which of the following?

Anemia, hyperkalemia, metabolic acidosis, and hyperphosphatemia occur in chronic glomerulonephritis.

A patient has undergone a renal biopsy. After the test, while the patient is resting, the patient complains of severe pain in the back, arms, and shoulders. Which of the following appropriate nursing interventions should be offered by the nurse?

Asses the patient's back and shoulder areas for signs of internal bleeding.

A client undergoes dialysis as a part of treatment for kidney failure, and is administered heparin during dialysis to achieve therapeutic levels. Which step should the nurse take to allow heparin to be metabolized and excreted in the client?

Avoid administering injections for 2 to 4 hours after heparin administration.

The nurse admits a client from a MVA. Which assessment could indicate possible renal trauma?

Hematuria

The nurse cares for a client after extensive abdominal surgery. The client develops an infection that is treated with IV gentamicin. After 4 days of treatment, the client develops oliguria, and laboratory results indicate azotemia. The client is diagnosed with acute tubular necrosis and transferred to the ICU. The client is hemodynamically stable. Which dialysis method would be most appropriate for the client?

Hemodialysis

Which is a prodromal symptom of hemolytic uremic syndrome in pediatric patients?

Diarrheal illness

The nurse is providing supportive care to a client receiving hemodialysis in the management of acute renal failure. Which statement from the nurse best reflects the ability of the kidneys to recover from acute renal failure?

The kidneys can improve over a period of months, may take 3-6 months

During the diuresis period of acute kidney injury (AKI), the nurse should observe the client closely for what complication?

Dehydration

A chronic renal failure client complains of generalized bone pain and tenderness. Which assessment finding would alert the nurse to an increased potential for the development of spontaneous bone fractures?

Hyperphosphatemia

Which nursing intervention should the nurse caring for the client with pyelonephritis implement?

Teach client to increase fluid intake up to 3 liters per day

The nurse is caring for a patient following an SCI who has a halo device in place. The patient is preparing for discharge. Which of the following statements made by the patient indicates the need for further instruction? a) "I will change the vest liner periodically." b) "I can apply powder under the liner to help with sweating." c) "If a pin becomes detached, I'll notify the surgeon." d) "I'll check under the liner for blisters and redness."

"I can apply powder under the liner to help with sweating." Explanation: The areas around the four pin sites of a halo device are cleaned daily and observed for redness, drainage, and pain. The pins are observed for loosening, which may contribute to infection. If one of the pins becomes detached, the head is stabilized in a neutral position by one person while another notifies the neurosurgeon. The skin under the halo vest is inspected for excessive perspiration, redness, and skin blistering, especially on the bony prominences. The vest is opened at the sides to allow the torso to be washed. The liner of the vest should not become wet because dampness causes skin excoriation. The liner should be changed periodically to promote hygiene and good skin care. Powder is not used inside the vest because it may contribute to the development of pressure ulcers.

The nurse is caring for a patient after kidney surgery. What major danger should the nurse closely monitor for?

Hypovolemic shock caused by hemorrhage

PP Questions: A nephrostomy tube may be inserted for which reason?

Large kidney stone

Which nursing intervention should the nurse caring for the client with pyelonephritis implement?

Teach client to increase fluid intake up to 3 liters per day.

Nicholas Pendergast, a 57-year-old professional athlete, has a lengthy history with chronic pyelonephritis. He is attending his annual physical and has become rather concerned with his potential for renal failure. How many clients with a history of chronic pyelonephritis require dialysis?

10% - 15%

A patient with chronic kidney failure experiences decreased levels of erythropoietin. What serious complication related to those levels should the nurse assess for when caring for this patient?

Anemia

A client is admitted for treatment of chronic renal failure (CRF). The nurse knows that this disorder increases the client's risk of: A. an increased serum calcium level secondary to kidney failure. B. water and sodium retention secondary to a severe decrease in the glomerular filtration rate. C. metabolic alkalosis secondary to retention of hydrogen ions. D. a decreased serum phosphate level secondary to kidney failure.

B. water and sodium retention secondary to a severe decrease in the glomerular filtration rate.

A client is admitted for treatment of chronic renal failure (CRF). The nurse knows that this disorder increases the client's risk of: a) a decreased serum phosphate level secondary to kidney failure. b) an increased serum calcium level secondary to kidney failure. c) water and sodium retention secondary to a severe decrease in the glomerular filtration rate. d) metabolic alkalosis secondary to retention of hydrogen ions.

C) water and sodium retention secondary to a severe decrease in the glomerular filtration rate. Explanation: The client with CRF is at risk for fluid imbalance — dehydration if the kidneys fail to concentrate urine, or fluid retention if the kidneys fail to produce urine. Electrolyte imbalances associated with this disorder result from the kidneys' inability to excrete phosphorus; such imbalances may lead to hyperphosphatemia with reciprocal hypocalcemia. CRF may cause metabolic acidosis, not metabolic alkalosis, secondary to inability of the kidneys to excrete hydrogen ions.

The nurse is preparing an education program on risk factors for kidney disorders. Which of the following risk factors would be inappropriate for the nurse to include in the teaching program? a) Pregnancy b) Diabetes mellitus c) Neuromuscular disorders d) Hypotension

D) Hypotension Hypertension, not hypotension, is a risk factor for kidney disease.

A nurse is reviewing the laboratory test results of a client with renal disease. Which of the following would the nurse expect to find? a) Decreased blood urea nitrogen (BUN) b) Decreased potassium c) Increased serum albumin d) Increased serum creatinine

D) Increased serum creatinine In clients with renal disease, the serum creatinine level would be increased. The BUN also would be increased, serum albumin would be decreased, and potassium would likely be increased.

A nurse assesses a client shortly after living donor kidney transplant surgery. Which postoperative finding must the nurse report to the physician immediately? a) Temperature of 99.2° F (37.3° C) b) Urine output of 20 ml/hour c) Serum sodium level of 135 mEq/L d) Serum potassium level of 4.9 mEq/L

Urine output of 20 ml/hour Explanation: Because kidney transplantation carries the risk of transplant rejection, infection, and other serious complications, the nurse should monitor the client's urinary function closely. A decrease from the normal urine output of 30 ml/hour is significant and warrants immediate physician notification. A serum potassium level of 4.9 mEq/L, a serum sodium level of 135 mEq/L, and a temperature of 99.2° F are normal assessment findings.

Select all of the following which are true about extracorporeal shock wave lithotripsy (ESWL).

• Stones are shattered into smaller particles that are passed from the urinary tract. • ESWL is administered with the client in a water bath or surrounded by a soft cushion while under light anesthesia or sedation.

A male client has doubts about performing peritoneal dialysis at home. He informs the nurse about his existing upper respiratory infection. Which of the following suggestions can the nurse offer to the client while performing an at-home peritoneal dialysis? a) Perform deep-breathing exercises vigorously. b) Avoid carrying heavy items. c) Auscultate the lungs frequently. d) Wear a mask when performing exchanges.

D) Wear a mask when performing exchanges The nurse should advise the client to wear a mask while performing exchanges. This prevents contamination of the dialysis catheter and tubing, and is usually advised to clients with upper respiratory infection. Auscultation of the lungs will not prevent contamination of the catheter or tubing. The client may also be advised to perform deep-breathing exercises to promote optimal lung expansion, but this will not prevent contamination. Clients with a fistula or graft in the arm should be advised against carrying heavy items.

A client who suffered hypovolemic shock during a cardiac incident has developed acute renal failure. Which is the best nursing rationale for this complication?

Decrease in the blood flow through the kidneys

The nurse expects which of the following assessment findings in the client in the diuretic phase of acute renal failure?

The diuretic phase of acute renal failure is characterized by increased urine output, hypotension, and dehydration.

A history of infection specifically caused by group A beta-hemolytic streptococci is associated with which of the following disorders? a) Chronic renal failure b) Nephrotic syndrome c) Acute glomerulonephritis d) Acute renal failure

Acute glomerulonephritis Explanation: Acute glomerulonephritis is also associated with varicella zoster virus, hepatitis B, and Epstein-Barr virus. Acute renal failure is associated with hypoperfusion to the kidney, parenchymal damage to the glomeruli or tubules, and obstruction at a point distal to the kidney. Chronic renal failure may be caused by systemic disease, hereditary lesions, medications, toxic agents, infections, and medications. Nephrotic syndrome is caused by disorders such as chronic glomerulonephritis, systemic lupus erythematosus, multiple myeloma, and renal vein thrombosis.

An elderly client is being evaluated for suspected pyelonephritis and is ordered kidney, ureter, and bladder (KUB) x-ray. The nurse understands the significance of this order is related to which rationale?

Detects calculi, cysts, or tumors. Urinary obstruction is the most common cause of pyelonephritis in the older adult. A KUB may reveal obstructions

During hemodialysis, toxins and wastes in the blood are removed by which of the following? a) Osmosis b) Diffusion c) Filtration d) Ultrafiltration

Diffusion Explanation: The toxins and wastes in the blood are removed by diffusion, in which particles move from an area of higher concentration in the blood to an area of lower concentration into the dialysate.

Which phase of acute renal failure signals that glomerular filtration has started to recover? a) Oliguric b) Recovery c) Diuretic d) Initiation

Diuretic Explanation: The oliguric period is accompanied by an increase in the serum concentration of wastes such as urea, creatinine, organic acids, and the electrolytes potassium, phosphorous, and magnesium. The initiation period begins with the initial insult and ends when cellular injury and oliguria develops. The diuretic phase is marked by a gradual increase in urine output, which signals that glomerular filtration has started to recover. The recovery period signals the improvement of renal function and energy level and may take 6 to 12 months.

A client is in end-stage chronic renal failure and is being added to the transplant list. The nurse explains to the client how donors are found for clients needing kidneys. Which statement is accurate?

Donors are selected from compatible living donors. R: pt w/ htn, malignant disease, or diabetes are excluded from the list.

The nurse is providing supportive care to a client receiving hemodialysis in the management of acute renal failure. Which statement from the nurse best reflects the ability of the kidneys to recover from acute renal failure?

The kidneys can improve over a period of months.

A nurse assesses a client shortly after living donor kidney transplant surgery. Which postoperative finding must the nurse report to the physician immediately?

Urine output of 20 ml/hour

The nurse instructs a client to perform continuous ambulatory peritoneal dialysis correctly at home. Which educational information should the nurse provide to the client?

Use an aseptic technique during the procedure.

Which of the following findings in the patient who has sustained a head injury indicate increasing intracranial pressure (ICP)? a) Increased pulse b) Widened pulse pressure c) Decreased body temperature d) Decreased respirations

Widened pulse pressure Explanation: Signs of increasing ICP include slowing of the heart rate (bradycardia), increasing systolic blood pressure, and widening pulse pressure (Cushing's reflex). As brain compression increases, respirations become rapid, the blood pressure may decrease, and the pulse slows further. This is an ominous development, as is a rapid fluctuation of vital signs. The temperature is maintained at less than 38°C (100.4°F). Tachycardia and arterial hypotension may indicate that bleeding is occurring elsewhere in the body.

The nurse is administering calcium acetate (PhosLo) to a patient with end-stage renal disease. When is the best time for the nurse to administer this medication?

With food. R: risk of hypercalcemia. They need to bind with dietary phosphorus in the GI tract.

A

6. A patient has AKI with a negative nitrogen balance. How much weight does the nurse expect the patient to lose? a. 0.5 kg/day b. 1.0 kg/day c. 1.5 kg/day d. 2.0 kg/day

C

15. A patient is placed on hemodialysis for the first time. The patient complains of a headache with nausea and begins to vomit, and the nurse observes a decreased level of consciousness. What does the nurse determine has happened? a. The dialysis was performed too rapidly. b. The patient is having an allergic reaction to the dialysate. c. The patient is experiencing a cerebral fluid shift. d. Too much fluid was pulled off during dialysis.

The nurse cares for a client with a right-arm arteriovenous fistula (AVF) for hemodialysis treatments. Which nursing action is contraindicated? A. Palpating the fistula for a "thrill" B. Obtaining a blood pressure reading from the right arm C. Placing the client's watch on the left wrist D. Obtaining blood samples from the left arm

B. Obtaining a blood pressure reading from the right arm

A client with decreased renal function is to receive a low-protein diet. The client asks the nurse why he needs this type of diet. The nurse would incorporate which reason into the response? A. Improve blood circulation B. Increase speed of treatment C. Improve digestion D. Lessen workload on the kidneys

D. Lessen workload on the kidneys

The nurse is completing a routine urinalysis using a dipstick. The test reveals an increased specific gravity. The nurse should suspect which condition?

Decreased fluid intake

Which of the following is as integumentary manifestation of chronic renal failure?

Gray brown skin color

Patients with urolithiasis need to be encouraged to:

Increase their fluid intake so that they can excrete 2.5 to 4 liters every day.

The nurse cares for a client with acute kidney injury (AKI). The client is experiencing an increase in the serum concentration of urea and creatinine. The nurse determines the client is experiencing which phase of AKI?

Oliguria

A nurse is reviewing the history of a client who is suspected of having glomerulonephritis. Which of the following would the nurse consider significant?

Recent history of streptococcal infection

A male client has doubts about performing peritoneal dialysis at home. He informs the nurse about his existing upper respiratory infection. Which of the following suggestions can the nurse offer to the client while performing an at-home peritoneal dialysis?

Wear a mask when performing exchanges.

The nurse is caring for a patient with ESKD. Which of the following acid-base imbalances is associated with this disorder?

pH 7.20, PaCO2 36, HCO3 14-

One of the roles of the nurse in caring for clients with chronic renal failure is to help them learn to minimize and manage potential complications. This would include:

restricting sources of potassium usually found in fresh fruits and vegetables.

A client comes to the Emergency Department complaining of a sudden onset of sharp, severe flank pain. During the physical examination, the client indicates that the pain, which comes in waves, travels to the suprapubic region. He states, "I can even feel the pain at the tip of my penis." Which of the following would the nurse suspect?

urainary calici


Related study sets

ch 58, 59, 60, 72 lehne & porth ch 33 nurs 223 test 3

View Set

Forensic Science Death and Decomposition

View Set

DV400 Development in Theory and Practice

View Set

Management of Patients with Hematologic Neoplasms

View Set